Космос и астрономия


Ответить в тред Ответить в тред

Check this out!
<<
Назад | Вниз | Каталог | Обновить тред | Автообновление
507 43 145

Тред тупых вопросов №113 Cecilia edition Аноним 23/09/19 Пнд 01:34:31 5140001
cecilia-payne-g[...].jpg (32Кб, 500x300)
500x300
solarspectrum.png (107Кб, 758x253)
758x253
Тред вопросов о жизни, Вселенной и всём таком.

Спрашиваем то, за что в других местах выдают путёвку в биореактор. Здесь анонимные учёные мирового уровня критически рассмотрят любые гениальные идеи и нарисованные в Paint схемы.

Предыдущий тут: >>510223 (OP)
https://2ch.hk/spc/res/510223.html

Q: Можно быстрее?
A: Можно упасть в пузырь альбукерке, наса уже почти надула его.

Q: Я начитался охуительных историй про уфологию, че делать, нам жопа?
A: Да, тебе жопа, можешь сгонять в зогач или куда оттуда пошлют.

Q: Что будет с человеком в вакууме без скафандра / если он упадет на черную дыру / попробует ступить на поверхность газового гиганта/солнца?
A: Он умрёт.

Q: Почему бы не привязать ракету к воздушному шару или стартовать с горы?
A: Космос - это не как высоко, а как быстро, большая часть энергии ракеты уходит на разгон вбок.
Подробнее тут https://what-if.xkcd.com/58/ (английский) https://chtoes.li/orbital-speed/ (перевод)
Аноним 23/09/19 Пнд 03:46:09 5140422
Аноним 23/09/19 Пнд 06:25:09 5140473
>>514000 (OP)
Во первых ебаный стыд, абулькерка!
Во вторых это надувателтство.
В третьих нельзя допускать интерференции, иначи придется идти пешком.
Аноним 23/09/19 Пнд 14:00:27 5141374
Меняется ли состав атмосферы с набором высоты, и если да то как
Аноним 23/09/19 Пнд 14:22:15 5141545
>>514137
Растет число легких молекул (водорода и гелия). На самом верху атмосферы, на высотах выше 200 км уже очень горячо около 1500 градусов и под действием солнечной радиации молекулы начинают ионизироваться. Вообще там пиздец происходит, лучше туда не ходи.
Аноним 23/09/19 Пнд 18:55:53 5143086
что мешает людям сейчас полететь на Марс? Слышал что лететь нужно всего 9 мес.
Аноним 23/09/19 Пнд 19:24:04 5143157
>>514308
Технически - ничего принципиально не мешает. Просто посадить живого человека на Марс человечеству сейчас по силам.
Но не так уж сильно это человечеству нужно. А развлечение страшно дорогое.
Аноним 23/09/19 Пнд 19:53:26 5143288
UzacUAc08Yw.jpg (241Кб, 493x750)
493x750
>>514000 (OP)
Зачем тратить деньги на космонавтов, решать вопросы со снаряжением, припасами, запасами кислорода? Еще радиация, а это уже весомый повод, чтобы не рыпаться с родной планеты.
Если даже цель всего этого пилить бабос, то делать это более прагматично и с большей эффективностью можно за счет беспилотных космических летательных аппаратов.
Технологии уже сейчас позволяют собирать ресурсы на Луне и в поясе астероидов.
Ядерные космические двигатели изобретены, а вместо этого все мозгуют, как бы нам на Марс обосраться цель, красная пыльная пустыня высадиться.


Аноним 23/09/19 Пнд 20:02:04 5143399
>>514328
изобретены не значит реализованы на практике
Аноним 23/09/19 Пнд 20:22:49 51434810
Не нашел проектов по искусственным горам кроме курортной в Голандии. Почему не навали кучу земли скажем в 20-40-60 и т.д. км. и от туда стартовать ракетами или ещё чем нибудь. Нашел такие проекты как Космический фонтан, Пусковая петля, Космический мост да тот же лифт, но вот искусственную гору даже близко. На вики было просто упоминание, но нечего конкретного. Чё за фигня
Аноним 23/09/19 Пнд 20:28:30 51434911
>>514348
См. ОП-пост:
> Q: Почему бы не привязать ракету к воздушному шару или стартовать с горы?
> A: Космос - это не как высоко, а как быстро, большая часть энергии ракеты уходит на разгон вбок.
> Подробнее тут https://what-if.xkcd.com/58/ (английский) https://chtoes.li/orbital-speed/ (перевод)
Аноним 23/09/19 Пнд 20:31:38 51435012
Gavni iz /b
Аноним 23/09/19 Пнд 20:33:35 51435113
>>514315
а разве тут можно решать с точки зрения выгоды и эффективности? Тут скорее другие критерии оценки. И лететь нужно только лишь потому что уже можно. А не потому что это все не окупится. Высадиться на другой планете это новый этап развития цивилизации, который навсегда войдет в историю Земли.
Аноним 23/09/19 Пнд 20:49:00 51435614
>>514351
> а разве тут можно решать с точки зрения выгоды и эффективности?
Как видишь, можно: именно так обычно решают адекватные руководители (всякие там президенты стран и проч.).
Выгода бывает не только грубо материальная. Многое в космосе было сделано ради престижа, например. В долговременных научных целях. И так далее.
Но просто для того, чтоб порадовался анон с двача, огроменные средства и силы вбухивать не будут.
Аноним 23/09/19 Пнд 21:36:46 51437315
Как чёрная дура может сохранять заряд (пусть и краткое время)? Ведь информация о наличии заряда не сможет выйти из-под горизонта. Или это связано с тем, что для внешнего наблюдателя вещество под горизонт не проваливается, а застывает очень близко к нему, и поэтому может ещё быть источником поля?
Аноним 23/09/19 Пнд 21:54:07 51437516
Кхм блеат
Аноним 23/09/19 Пнд 22:31:55 51438817
1.jpg (55Кб, 1300x650)
1300x650
>>514000 (OP)
Это не она про Звезду Смерти презентацию делала?
Аноним 23/09/19 Пнд 23:53:35 51440318
>>514373
Краткое.
Она потом из говен вокруг собирает обратный заряд и нейтральной становится.
Аноним 24/09/19 Втр 00:54:57 51440919
>>514328
>Зачем тратить деньги на космонавтов, решать вопросы со снаряжением, припасами, запасами кислорода?

Это многогранный вопрос. Люди начали летать в прошлом веке, еще тогда они понимали, что на ракетах с химическим двигателем много не налетаешь, но все ждали новых двигателей или вообще даже новые физические принципы перемещение. Но ничего этого так и нет. Люди летают по инерции, типа раз Гагарин полетел, то и нам надо летать. А если сейчас перестать летать, то я даже не знаю что будет, может вообще будет психологическая травма у общества и на космос все забьют болт.

>Технологии уже сейчас позволяют собирать ресурсы на Луне и в поясе астероидов.

Это бессмысленно. На Земле есть все нужные ресурсы и добывать их тут гораздо дешевле, чем в космосе.

>Ядерные космические двигатели изобретены

Технически его реализовать не получается. В теории он работает, а вот на практике нет. Ядерные двигатели это похоже тупиковая ветвь.
Аноним 24/09/19 Втр 00:56:03 51441020
>>514351
>а разве тут можно решать с точки зрения выгоды и эффективности

Все дорогие проекты оцениваются с точки зрения экономики. Иначе нельзя.
Аноним 24/09/19 Втр 00:56:38 51441121
>>514373
>Как чёрная дура может сохранять заряд
Вероятно у черной дыры нет никакого заряда.
Аноним 24/09/19 Втр 03:09:51 51443122
>>514411
https://en.wikipedia.org/wiki/Charged_black_hole
В теории никто не мешает ей быть, на деле же вряд ли.
Я бы даже вспизднул что черная дыра гипотетический объект, но мы недавно, говорят, сфоткали одну.
Аноним 24/09/19 Втр 03:24:27 51443323
>>514431
>В теории никто не мешает ей быть, на деле же вряд ли.
Соглашусь.
>Я бы даже вспизднул что черная дыра гипотетический объект, но мы недавно, говорят, сфоткали одну.
Ну мы не "видели" ее горизонта, астрофизики говорят, что нужны чувствительные гравитационные телескопы и космический радиоинтерферометр с как можно большей базой, чтобы позыкать что там за "поверхность" и как ведут себя гравитационные волны, по их "поведению" можно понять что у нее с поверхностью и есть ли она.
Аноним 24/09/19 Втр 05:00:04 51449324
>>514000 (OP)
Сомневаюсь, что тут кто-нибудь знает тетеньку с оппика, несмотря на то, что

Cecilia was able to accurately relate the spectral classes of stars to their actual temperatures by applying the ionization theory developed by Indian physicist Meghnad Saha. She showed that the great variation in stellar absorption lines was due to differing amounts of ionization at different temperatures, not to different amounts of elements. She found that silicon, carbon, and other common metals seen in the Sun's spectrum were present in about the same relative amounts as on Earth, in agreement with the accepted belief of the time, which held that the stars had approximately the same elemental composition as the Earth. However, she found that helium and particularly hydrogen were vastly more abundant (for hydrogen, by a factor of about one million). Her thesis concluded that hydrogen was the overwhelming constituent of stars (see Metallicity), making it the most abundant element in the Universe.

Ну и на то, что она была first woman to be promoted to full professor from within the faculty at Harvard's Faculty of Arts and Sciences
Аноним 24/09/19 Втр 05:15:32 51450825
>>514308
человек наверное не выдержит такое долгое пребывание в условиях невесомости. Обратно уже почти точно не доберётся и скончается довольно скоро после прибытия на Марсе - если вообще сможет оказаться на поверхности
Ещё технические проблемы с приземлением(примарсение?) на марсианскую поверхность, из-за очень низкой плотности атмосферы. Пилотируемый корабль как марсоход приземлить на Марс нельзя
Аноним 24/09/19 Втр 05:31:21 51452226
>>514508
>Пилотируемый корабль как марсоход приземлить на Марс нельзя
Почему?
Аноним 24/09/19 Втр 08:29:55 51454727
>>514410
>иначе нельзя
Можно.
Аноним 24/09/19 Втр 09:48:48 51457028
>>514403
Ну а если говна вокруг почти не будет, то долго будет заряженной - не в этом суть. Почему поле сразу не исчезает, хотя заряд уже провалился? Вон говорят исчезноевение инфы в чд, а заряд к этой инфе схуяли не относится?
Аноним 24/09/19 Втр 10:00:39 51457929
>>514570
Обожди, тут вопрос поважнее.
А с хуев ли это проблема какая-то, это ваше исчезновение инфы? Ну исчезла, ну и хуй с ней, какая разница?
Вон она испарится через квадриллион лет кварками рандомными, в чем проблема?
Аноним 24/09/19 Втр 10:16:15 51458430
>>514579
Потому что тогда её строение и процессы невозможно описать в рамках квантмеха, ото же даёт безосновательную экстраполяцию, что вся масса - в бесконечно плотной точке/кольце. Это никак ни смоделировать, ни проверить, вообще никаких подходов. Т.е. чд не поддаётся научному подходу, не вписывается в научную картину мира, неконструктивная смесь математической абстрактности и суеверных домыслов. Любая внятная теория будет предпочтительней.
Аноним 24/09/19 Втр 11:04:14 51459931
>>514584
Все равно непонятно в чем проблема.
Вон у нас БВ случился из какой-то математической абстракции маленькой точки, вселенная заспавнитась таким образом.
Как заспавнилась так и распидорасится черной дырой.
Масса сохраняется. А масса это энергия. Какая вам еще информация нужна?
Аноним 24/09/19 Втр 12:37:41 51464032
Как солнечный спектр может иметь линии поглощения металлов, если они все утонули в ядро, а фотоны из ядра миллиарды триллионов раз поглощаются-излучаются, прежде чем в космос улететь?
Аноним 24/09/19 Втр 12:47:36 51464533
>>514599
>Все равно непонятно в чем проблема.
Проблема в СИНГУЛЯРНОСТИ. Эту ахинею астрофизикам подкинули математики и астрофизики на нее клюнули. Когда математик не может свои долбанные уравнения решить, то он все валит на сингулярность, дескать тут ничего решить нельзя, возникает ебаная сингулярность.
Аноним 24/09/19 Втр 12:49:14 51464634
>>514599
>Вон у нас БВ случился из какой-то математической абстракции маленькой точки
БВ это гипотеза. Есть еще гипотеза столкновения "бран" (многомерных хуиток в теории струн). Дескать одна брана "упала" на другую и моментально везде случился БВ и никакой тебе маленькой точки.
Аноним 24/09/19 Втр 12:50:03 51464735
>>514640
>Как солнечный спектр может иметь линии поглощения металлов, если они все утонули в ядро,
С чего бы они утонули? Они же не жидкие, это мать его плазма!
Аноним 24/09/19 Втр 12:59:23 51464936
>>514647
Потому что тяжелее, гравитация, все дела, и времени дохуя на это было. Или, может, частично тонут, но как тогда смогли точно определять состав звезды в целом? Если на солнце аккуратно положить астероид (ну чтоб он не погрузился в него под действием импульса), его вещество так и будет болтаться у поверхности миллионы столетий?
Аноним 24/09/19 Втр 13:10:41 51465437
>>514649
>Потому что тяжелее, гравитация
Компенсируется давлением излучения. Когда излучение станет слабее и гравитация победит, то Солнцу придет каюк.

>Если на солнце аккуратно положить астероид (ну чтоб он не погрузился в него под действием импульса), его вещество так и будет болтаться у поверхности миллионы столетий?

На поверхности Солнца наблюдается значительная конвекция, там все попердывает и постоянно перемешивается поэтому эта часть Солнца называется конвективной зоной.
Аноним 24/09/19 Втр 13:40:32 51466938
>>514348
>Почему не навали кучу земли скажем в 20-40-60 и т.д. км
Во-первых, это ебически дорого. Во-вторых, твоя куча земли просто рассыплется под своим весом. Предел прочности гранитов под собственным весом - что-то около десятка километров вроде, как раз чуть больше самых высоких гор. Дальше неизбежно начнется разрушение. Да и насыпать гранит как-то сложно, максимум щебень (ну ты понел) или бетон, но бетону нужно много времени на застывание. Ну и в-третьих, как уже сказали, это не такой уж гигантский выигрыш для вывода на орбиту.
Аноним 24/09/19 Втр 14:58:25 51469639
>>514654
> Компенсируется давлением излучения. Когда излучение станет слабее и гравитация победит, то Солнцу придет каюк.
Это тут причём? Земле давление тоже не даёт в чёрную дуру превратиться, что не помешало почти всем тяжёлым элементам в ядро утонуть. Почему в солнце так же не тонут? Или тонут, но каким образом излучают тогда?
Аноним 24/09/19 Втр 15:15:26 51470140
>>514696
>не помешало почти всем тяжёлым элементам в ядро утонуть.
Но ведь у них там ядерная реакция начнется?
Аноним 24/09/19 Втр 15:16:08 51470241
>>514696
>Это тут причём?
При том, чо в центре Солнца идут ядерные реакции которые производят много излучения поток которого так пиздато греет меня сейчас.

>Земле давление тоже не даёт в чёрную дуру превратиться

Это ДРУГОЕ. На Солнце не давление вещества не дает ему схлопнуться, а давление излучения, потока фотончиков из центра Солнца. А почему металлы не утонули так потому что масса Солнца неебическая и слой над ядром окружен слоем неебичеки сжатого давлением водорода и гелия, там все настолько плотно сжато как, что ты даже представить себе не можешь, настолько сжато что никакого перемешивания вещества не происходит. Вот скажи, ты видел когда-нибудь водородную плазму под давлением имеющую плотность вольфрама? Вот и какие тебе металлы будут тонуть в такой плазме?
Аноним 24/09/19 Втр 15:44:54 51471042
>>514349
>Космос - это не как высоко, а как быстро, большая часть энергии ракеты уходит на разгон вбок.

значит нужно разгонять ракеты на рельсах и запускать как бы горизонтально?
Аноним 24/09/19 Втр 15:46:18 51471343
>>514710
Надо взрывать паровозы.
Аноним 24/09/19 Втр 15:53:08 51471844
image.png (32Кб, 1875x862)
1875x862
>>514710
Собственно, вот иллюстрация. В чем проблема такое заебошить?
Аноним 24/09/19 Втр 16:32:19 51472645
>>514718
ракета размером с великобританию судя по масштабу?
Аноним 24/09/19 Втр 16:38:16 51472846
>>514702
Ну конвективная зона мешает же вещество. А в больших звёздах и ядра вроде конвективные. И несмотря на то, что всё вещество перемешивается, для переноса вверх тяжелых ядер будет нужно больше энергии и они, в среднем, будут ниже концентрироваться... по логике.
Аноним 24/09/19 Втр 17:13:46 51473747
Что почитать про химию космического пространства? Конкретно про пыль, газ, поверхности астероидов, комет итд. Там вроде много чего интересного происходит, даже всякая органика образуется. Можно на английском.
Аноним 24/09/19 Втр 17:36:23 51474848
>>514726
если бы я нарисовал в масштабе, то чтоб увидеть ракету, тебе бы пришлось доставать микроскоп, который ты используешь когда хочешь увидеть свой член
стоит ли дарить парню-физику подвеску из настоящего метеорита? Аноним 24/09/19 Втр 18:40:21 51475849
15452533255c1ab[...].jpg (64Кб, 666x1000)
666x1000
у моего молодого человека скоро день рождения. хочу ему приподнести такой необычный подарок, но я немного боюсь что он обидется т.к подумает что метеорит это в целом простая железяка и мол только деньги зря потратила. скажите пожалуйста, имеют ли метеориты какую-либо ценность, уникальность в плане состава? спасибо.
Аноним 24/09/19 Втр 18:52:49 51476050
>>514758
Подари ему хороший отсос и приведи подругу
Аноним 24/09/19 Втр 18:57:34 51476151
Аноним 24/09/19 Втр 18:58:46 51476252
>>514760
А подругу зачем? Подруга - куколд?
Аноним 24/09/19 Втр 19:13:31 51476653
Аноним 24/09/19 Втр 21:41:24 51480154
>>514726
>>514748
тогда нахрена ты землю вниз загнул, идиот?
Аноним 24/09/19 Втр 21:50:54 51480555
Может ли звезда ебнуть без остатка ? Чтобы одно облако газа осталось
Аноним 24/09/19 Втр 21:52:53 51480656
Аноним 24/09/19 Втр 22:17:56 51481057
>>514758
В метеорите могут содержаться споры инопланетных микроорганизмов. Это опасно.
Аноним 24/09/19 Втр 22:40:20 51481358
>>514801
ты пукнутый или что?
Аноним 24/09/19 Втр 22:59:10 51481659
>>514718
> В чем проблема
Да как же вы заебали. "В чем проблема построить амбар высотой 50 км, мы же можем построить 50-метровый амбар, давайте просто увеличим его в 1000 раз?" В квадрате-кубе основная проблема, а так же в том, что это только по объему работ миллиард амбаров, блядь.

Ты не можешь просто пропорционально увеличить что-то в N раз и ожидать, что оно по-прежнему будет работать. Как минимум давление на основание вырастет в N раз (масса в кубе, площадь основания - в квадрате). А значит 1) основание разрушится, ну разве что ты его из цельных нанотрубок сделаешь; 2) вся эта хуйня продавит земную кору и утонет в мантии. А еще, ты ожидаешь, что человечество резко начнет производить кубокилометры стройматериалов для твоего очередного мегапроекта? На какие шиши? Из чего? Какими мощностями? Лень считать, но подозреваю, что если собрать все постройки человечества за всю историю, они окажутся меньше этой ебалы.
Аноним 24/09/19 Втр 23:20:48 51481860
Аноним 24/09/19 Втр 23:21:37 51481961
image.png (19Кб, 1212x542)
1212x542
image.png (18Кб, 658x286)
658x286
>>514816
>Ты не можешь просто пропорционально увеличить что-то в N раз и ожидать, что оно по-прежнему будет работать

Ты тоже пукнутый. Что мешает построить типа эстакаду высотой 50 метров и длинной 25. Это как мост только легче потому что не в воде. Такие параметры будут у хуйни которая идет ровно горизонтально и не загибается. А можно и выше/длиннее
Аноним 24/09/19 Втр 23:32:27 51482162
>>514819
Ничего не мешает. Кроме того, что в этом нет смысла, потому что твоя ракета сразу же разуплотнится, когда попробует набрать 8 км/c в атмосфере околоземной плотности. Твоя хуйня на уровне Земли никакого смысла не имеет. Да и строго горизонтальной ей быть не надо. А там, где смысл есть - как минимум километров 10, на глазок (может и 30), - она станет тех самых циклопических и совершенно недостижимых размеров, которые угадываются по твоей картинке первой картинке.
Аноним 25/09/19 Срд 02:44:13 51483563
image.png (401Кб, 1045x1634)
1045x1634
>>514737
Гуглится на раз, но как-то непонятно что тебе конкретно охота, тема обширная же.
Ну эта, если именно кометы 9783642748073
А так - водород да гелиум.
Потом звезды ХУЯК - и вся таблица менделеева.
А потом молекулы воды, аммиака и прочих простых соединений и все заверте...
Аноним 25/09/19 Срд 02:46:53 51483664
image.png (31Кб, 350x131)
350x131
>>514805
Нет.
СОВСЕ МАЛАЯ не ебнет, а потухнет.
МАЛАЯ ебнет в белого карлана.
БОЛША ебнет в нейтрониевую звезду.
а ОСНЕ БОЛЬШАЯ в чорную дырень.

>>514806
С чего ты взял-то?
Аноним 25/09/19 Срд 02:50:17 51483765
>>514821
Че вы сразу ему так не ответили-то?

>>514819
Такая хуита имеет смысл на безатмосферных телах типа луны. Там, собственно, куча фантазеров и фантастов разных мастей и предлагают строить ЭМ-ускоритель, т.к. нет никаких проблем выйти на орбиту на высоте 10 метров (главное потом пропукайся еще повыше, чтоб не ебнуться назад)
Проверить можешь в огурцаче просто разогнавшись быстро на колесиках по поверхности моря Минмуса, например. Очень наглядно. Альцо поймешь чому такая ебала на земляшке не канает.
Аноним 25/09/19 Срд 03:01:10 51483966
>>514837
Да там много проблем. Например, для достижения 8 км/c даже при 10 g нужно 320 км, а вовсе не 25 (опять циклоп). Или технические проблемы опоры на рампу (иначе смысл теряется) на такой скорости: трение, например. Да и вообще, все потери на сопротивление гравитации в районе 10-20% вроде, бОльшая часть дельты-в именно для разгона нужна. И тут рампа никак не поможет.
Аноним 25/09/19 Срд 03:38:36 51484267
>>514737
>Что почитать про химию космического пространства?
Для начала послухай и реши надо ли тебе оно. А то вдруг ты ничего не поймешь.

https://www.youtube.com/watch?v=yU-MnmLdeGg
Аноним 25/09/19 Срд 03:52:28 51484368
Не следуй совету этого анона >>514842
Русские видео не смотри ни в коем случае если английский знаешь. Только если не знаешь - смотри переводы.
Аноним 25/09/19 Срд 03:54:11 51484569
>>514843
>Русские видео не смотри ни в коем случае
Чому?
Аноним 25/09/19 Срд 03:56:31 51484670
>>514845
Почему-то слишком легко нарваться на антинаучную или просто бестолковую парашу. Сразу деза уровня РенТВ от уверенно звучащего пиздобола в уши польется.
Про химию каналов, увы, не знаю, не интересовался про космологию есть PBS Space Time, крайне рекомендую.
Аноним 25/09/19 Срд 04:00:26 51484971
>>514846
>Почему-то слишком легко нарваться на антинаучную или просто бестолковую парашу.
Согласен. Но Дмитрий Зигфридович Вибе доктор физико-математических наук и заведующий отделом физики и эволюции звёзд Института астрономии РАН. Так что не ссы, смотри смело.
Аноним 25/09/19 Срд 04:02:03 51485172
>>514846
>про космологию есть

Есть годнота на русском.

https://www.youtube.com/watch?v=DIKtrSj-QeU

https://www.youtube.com/watch?v=GDHxcxjUGAs

https://www.youtube.com/watch?v=jHbdd0P7Sh0

https://www.youtube.com/watch?v=OorGJOfVilw

https://www.youtube.com/watch?v=6adSnHzwuDk

https://www.youtube.com/watch?v=u4GDROaj4ls

https://www.youtube.com/watch?v=o8ghcQ31nxw

https://www.youtube.com/watch?v=ov76tpatbVY

https://www.youtube.com/watch?v=j2My_ieOsXs

https://www.youtube.com/watch?v=NsmIcSUi87U

https://www.youtube.com/watch?v=AaV8o9HDGz4

https://www.youtube.com/watch?v=SdIw2W6FHec
Аноним 25/09/19 Срд 07:31:56 51487873
>>514821
>когда попробует набрать 8 км/c
Кто тебе сказал что нуно набрать именно такую скорость? Мы не выстреливаем ракетой а даем ей некий стартовый разгон, допустим 3-4 км/с, чтоб меньше топлива тратить.
Аноним 25/09/19 Срд 07:37:30 51487974
>>514878
И она о плотную атмосферу останавливается нахуй, умник.
Аноним 25/09/19 Срд 07:53:49 51488175
А у России есть радиотелескопы миллиметрового диапазона?
Аноним 25/09/19 Срд 08:14:25 51488276
image.png (94Кб, 987x454)
987x454
Аноним 25/09/19 Срд 08:32:32 51488677
>>514882
Раз он есть, почему нет ни одной статьи о нем в Вики на других языках?
Аноним 25/09/19 Срд 08:39:38 51488978
image.png (262Кб, 1433x1206)
1433x1206
Аноним 25/09/19 Срд 08:50:20 51489279
>>514889
>Потому, что никто не запилил про него статью.
Почему раз он такой крутой? Про все крутые радиотелескопы статьи есть, а про этот почему-то нет?
Аноним 25/09/19 Срд 09:25:12 51489780
>>514879
Нет, потом она дальше на пердаковой тяге. просто будет стартовать не как сейчас с 0 км/с а с 3 км/с
Аноним 25/09/19 Срд 13:18:27 51496681
>>514839
Слыш, 8км/с это, внезапно, на 1/3 больше скорости звука в сталисели.
Из чего этот школ будет делать колёсья, рельсы, валы, подшипники и всё вот это вот?
Ды даже при лимите 3км/с в механизмах будут элементы с большей скоростью взаимодействия.
Аноним 25/09/19 Срд 13:20:45 51496782
>>514892
Ответ уже есть в посте: потому, что никто не запилил. Про много существующих телескопов нет отдельной статьи.
Аноним 25/09/19 Срд 13:23:51 51496983
>>514897
Ты понимаешь, что ты горишь на 10 махах летя близко к земле и тебе щас надо разворачиваться наверх иначе сгоришь нахуй. Или не понимаешь? Или кажется, что ты можешь так полчаса полететь сжигая топляк против торможения атмосферы чтоб наконец по горизонтали выбраться к менее плотным слоям?

Запусти огурцов, ей-богу, там наглядно понятно будет что к чему. Только поправку не забудь на то, что кербин - 1/10 земли и корабли чуть более волшебные чем ИРЛ.

Если доходит что атмосфера дохуя мешает и ты захочешь таки наверх подняться на малом газу и там потихоньку выбираться, то ты только что схему взлета Скайлона.
Аноним 25/09/19 Срд 13:40:47 51497384
>>514969
>Ты понимаешь, что ты горишь на 10 махах летя близко к земле
Можно лететь и абляцировать. Например.
Аноним 25/09/19 Срд 13:43:23 51497585
image.png (595Кб, 1184x492)
1184x492
>>514973
Зачем, во имя чего?
Аноним 25/09/19 Срд 13:45:03 51497686
>>514973
Ты так и не понимаешь сути, тебя весь путь тормозит атмосфера, даже если ты не сгораешь, ты кучу топлива впустую сжигаешь на поддержание скорости.
Аноним 25/09/19 Срд 13:50:44 51497787
>>514975
Ради лулзов, например.
Аноним 25/09/19 Срд 13:54:11 51497988
>>514976
> ты кучу топлива впустую сжигаешь на поддержание скорости.
А если мое топливо это атмосфэра?
Аноним 25/09/19 Срд 13:55:46 51498089
>>514977
Вот так и получаются НЛО. Никто не поверит что какой-то уебан сжег 8000 тонн керосина чтобы прочертить яркую прямую абляции и выхлопа над европейской частью страны.

>>514979
А я поэтому писал топливо. Если ты даже ГПВРД запряжешь и сможешь обуздать воздух и сжигать только топливо... ты все равно будешь сжигать дохуя топлива.

Сейчас слово за слово, торг за торг и до Скайлона договоримся, базарю.
Аноним 25/09/19 Срд 14:01:00 51498290
image.png (477Кб, 700x546)
700x546
>>514980
>до Скайлона договоримся
Ну это хуже всего
Аноним 25/09/19 Срд 14:02:32 51498691
>>514980
Ты не понял. Топливо это атомы азота и кислорода, которые путем нехитрых ядерных превращений трансмутируем в другие элементики и маем немного энергии.
Аноним 25/09/19 Срд 14:13:04 51500092
>>514982
Так неча было на трамплинах скакать, взлетал бы как белый человек на вертикальной ракете или хотя бы по петле Лофстрома карабкался, другой разговор был бы.

>>514986
Так что ж ты сразу не сказал. Заодно энергетический кризис решил, вот умница!
Аноним 25/09/19 Срд 14:19:59 51500893
>>515000
>Заодно энергетический кризис решил, вот умница!
И атмосфэрку вам немного подобновил.
Аноним 25/09/19 Срд 14:22:37 51501094
image.png (85Кб, 1100x620)
1100x620
>>515008
Это мы и сами умеем.
Аноним 26/09/19 Чтв 06:09:18 51528995
17tLVHy-fhU2yHB[...].jpeg (324Кб, 1350x901)
1350x901
Почему вселенная молчит?
26/09/19 Чтв 06:28:08 51529196
>>515289
потому же, почему
Аноним 26/09/19 Чтв 07:02:54 51529797
>>515289
Что понимается под "молчит"?
Она вовсю пиздит на всех длинах волн спектра.
Аноним 26/09/19 Чтв 07:06:18 51529898
>>515297
Имеется в виду наличие других разумных видов.
Аноним 26/09/19 Чтв 07:09:34 51529999
>>515298
>других разумных
>других
>разумных
Какая смешная обезьяна.
Аноним 26/09/19 Чтв 07:31:04 515301100
>>515298
Потому, что не нашли пока нигде.
Ну разве что дельфины и несколько штук приматов на этой планете есть из разумных.
Аноним 26/09/19 Чтв 07:37:47 515302101
Лингвистика космоса вопросы. Я думаю, они изысканно тупы.
Почему Меркурианский, Венерианский, Марсианский, итд, а не Венерский, Марсский, Юпитерский етц?
Как будет такое же прилагательное про Землю? Земельный? Земляной? Землианский?
Почему юпитерская орбита будет Jovian а не jupiterian?
Почему у нас не так?
Почему при этом marsian, а не aresian?

Я еще один вопрос хотел, но забыл.
Аноним 26/09/19 Чтв 07:38:57 515303102
>>515301
>на этой планете
Разумной жизни нет.
Аноним 26/09/19 Чтв 07:50:12 515306103
>>515303
Подожди, а как же Нил Тайсон, Митио Каку, Брайан Кокс, Дольф Лунгрен, Григорий Перельман, Барак Обама, Рик Роснер, Рик Санчез, Майкл Висос, Дерек Веритасиум, Твайлайт Спаркл, Хайден Кристенсен и Стивен Хокинг?
Аноним 26/09/19 Чтв 07:52:31 515308104
>>515302
>Как будет такое же прилагательное про Землю?

Грязный. Настоящая название нашей планеты - Грязь.
Аноним 26/09/19 Чтв 07:59:04 515311105
>>515306
Благодарю за иллюстрацию.
Аноним 26/09/19 Чтв 08:09:18 515319106
image.png (220Кб, 1024x768)
1024x768
>>515308
Не, грязный это обмазанный грязью, а не принадлежащий/относящийся к грязи.
Тогда грязевой уж.
Апоапсис грязевой орбиты МКС составляет 400 километров.
Во, так заебись.
Грязная орбита ж не звучит. Как будто в огурцаче глюк с орбитой словил.
Аноним 26/09/19 Чтв 08:11:35 515322107
Noice..mp4 (620Кб, 1280x720, 00:00:03)
1280x720
>>515311
Не за что. Я рад, что смог убедить тебя в наличии разумной жизни простым примером.
Аноним 26/09/19 Чтв 08:18:01 515324108
>>515322
>этот днотадаунский говноролик
Игроки в жопа-2 - исчерпывающее и окончательное доказательство.
Разумной жизни на этой планете нет.
Аноним 26/09/19 Чтв 08:27:43 515327109
>>515324
Обожди, то что днотапитухи используют что-то не значит что это их, этот мемос пошёл с форчка и YTP.
YTP - да, доказательство имбецильной жизни на этой планете.
Аноним 26/09/19 Чтв 08:31:37 515329110
>>515327
Значит.
Зашквар невозможно провернуть назад.
Аноним 26/09/19 Чтв 11:05:06 515371111
>>515324
Наличие одноклеточных - доказательство отсутствия многоклеточных?
Аноним 26/09/19 Чтв 13:48:55 515406112
Аноним 26/09/19 Чтв 17:42:52 515458113
>>515406
>>515324
>Да
Твой ответ — исчерпывающее и окончательное доказательство.
Отрицательных ответов на этой планете нет.
Аноним 26/09/19 Чтв 20:38:40 515507114
>>514154
>около 1500 градусов
Каких градусов?
Аноним 26/09/19 Чтв 22:15:02 515545115
>>515507
Градусов Делиля.
Аноним 26/09/19 Чтв 22:15:30 515546116
>>515302
Бамп. Почему за день никто не помог?
Аноним 26/09/19 Чтв 23:17:33 515554117
Аноним 26/09/19 Чтв 23:19:23 515557118
С какой высоты нужно упасть на Титане, чтобы разбиться нахуй?
Аноним 26/09/19 Чтв 23:25:15 515559119
>>515289
Пруфы есть что молчит? Ты сам проверял? Может там во всю ящерики с птаагами срутся на всех частотах.
Аноним 26/09/19 Чтв 23:27:18 515561120
Допустим, у нас есть кусок говна с консистенцией творога с массой Солнца, какие процессы в нем будут происходить и в какой последовательности? Как я понял, он начнет переплавливаться во что-то другое в центре и постепенно затронет все остальные слои? Через какое время говно полностью переработается?

Образование у меня 4 класса церковно-приходской, объясните с пояснениями пожалуйста.
Аноним 26/09/19 Чтв 23:28:42 515562121
>>515546
Потому что соси хуй, быдло
Аноним 26/09/19 Чтв 23:33:00 515564122
>>515557
Высота наверное обратно пропорциональна ускорению свободного падения. Вот у нас уже с трех метров больно падать, значит на Титане это будет 3 * 9.8 / 1.352 = 21.74556213017752 метров.
Аноним 26/09/19 Чтв 23:39:27 515566123
3 факта почему ты думаешь, что есть внеземная жизнь/ее нет
Аноним 26/09/19 Чтв 23:42:54 515567124
>>515561
Оно немного сожмется и внутри ебанет от поднявшейся температуры. Творог постепенно переработается в тяжелые элементы и станет ядром. Вся куча с течением времени будет грется, пока не раскалится и не будет светить.
Аноним 26/09/19 Чтв 23:52:38 515569125
>>515567
То есть можно создать говяное солнце и освещать планеты говяным светом? Ничего себе!
Аноним 26/09/19 Чтв 23:54:34 515571126
>>515566
1. Эволюция хуйня и не работает.
2. Предыдущего факта достаточно.
Аноним 26/09/19 Чтв 23:55:38 515573127
>>515569
Да. Пофиг из чего оно будет сделано, оно все равно будет светить.
Аноним 27/09/19 Птн 02:25:14 515587128
>>515566
Скорее всего жизнь есть и есть рядом т.е. в пределах нескольких сотен парсеков. Но то что она разумна кажется маловероятным.
Аноним 27/09/19 Птн 04:40:37 515600129
Аноним 27/09/19 Птн 04:41:39 515601130
>>515573
Не, не пофиг. Из железа не будет.
Аноним 27/09/19 Птн 06:11:55 515607131
>>515564
Так там же еще плотная атмосфера.
Аноним 27/09/19 Птн 06:49:59 515615132
>>515607
Ты думаешь, что там терминалка медленнее скорости при которой человек ломает ноги и он будет падать как в киселе?
Аноним 27/09/19 Птн 06:54:28 515618133
Какие есть комплунктерные игори в которых можно лампово посычевать на другой планетке?
Возраст игры не важен абсолютно, можно аж с зарождения 16-цветного графена.
Огурцы и бесчеловечное небо не предлагать.
Аноним 27/09/19 Птн 06:55:54 515619134
С какого размера камень не прилетит целиком в планетку а будет распидорашен на подлете?
Насколько большая йоба может вот так мимо земляшки прилететь, цепануть атмосферу и улететь, чутка поджарившись? Может ли это быть кото клизменно?
Аноним 27/09/19 Птн 06:58:33 515621135
>>515618
>Какие есть комплунктерные игори в которых можно лампово посычевать на другой планетке?
Doom 2, Doom 3, Doom Eternal (уже скоро).
Аноним 27/09/19 Птн 07:01:00 515622136
image.png (436Кб, 838x557)
838x557
>>515561
Самое ближайшее есть наглядно для тебя - это планетоид из хомячков кротов: https://what-if.xkcd.com/4/
На русской мове: https://chtoes.li/a-mole-of-moles/
Подробно смачно описано как их распидорасит и будет каша из фарша покрытая мягкой меховой оболочкой.
Аноним 27/09/19 Птн 07:01:44 515623137
4.jpg (28Кб, 400x325)
400x325
>>515619
>Может ли это быть кото клизменно?
Аноним 27/09/19 Птн 07:02:30 515624138
>>515621
>лампово посычевать
>Doom
Джон Сталверн ждал. Светильники над ним мигнули и искрнули в воздух. На базе были демоны. Он не видел их, но ожидал их годами. Его предупреждения Палковнеку Джосону не были услышаны, а сейчас уже слишком поздно. Давно уже слишком поздно, в любом случае.

Джон был космодесантником четырнадцать лет. Когда он был молодым, он смотрел на космолеты, и сказал папе "Я хочу быть на кораблях, папочка."

Папа сказал "Нет! Ты будешь УБИВАННЫЙ ДЕМОНАМИ"

Было время, когда он ему верил. Затем, года он постаршенел, он перестал. Но сейчас, на космической станции базы ОАК он знал что там были демоны. "Это Джосон" протрещало радио. "Ты должен драться с демонами!"

Так что Джон хватал свою палзменную винтовку и взорвал стену.

"ОН ХОЧЕТЬ УБИТЬ НАС" сказали демоны

"Я выстрелю в него" сказал кибердемон и он выстрелил ракетными снарядами. Джон плазмнул в него и попытался подорвал его. Но потом потолок упал и они были в ловушке и не могли убивать.

"Нет! Я должен убивать демонов" закричал он

Радио сказало "Нет, Джон. Ты и есть демоны"

И тогда Джон был зомби.
Аноним 27/09/19 Птн 07:03:16 515625139
>>515289
Потому, что звук не распространяется в вакууме.
Аноним 27/09/19 Птн 07:04:27 515626140
>>515624
Финал катастрофически прекрасен.
Аноним 27/09/19 Птн 07:06:22 515627141
>>515626
Недаром это один из почетнейших текстов в коричневом фонде копипаст про игры.
Аноним 27/09/19 Птн 07:06:28 515628142
>>515625
Чой-то? В космосе не везде вакуум, есть огромные газопылевые облака которые наполнены звуками, звуковыми ударными волнами которые создают неоднородности структуру облака и формируют звезды. В НАЧАЛЕ БЫЛО СЛОВО.
Аноним 27/09/19 Птн 07:07:26 515629143
>>515618
Ancestors: The Humankind Odyssey. Я вообще на нем сижу как на героине.
Аноним 27/09/19 Птн 07:08:43 515630144
image.png (24Кб, 500x47)
500x47
>>515629
>Я вообще на нем сижу как на героине
>Planned Release Date: 2020
Но как? Ты из будущего капчуешь?
Аноним 27/09/19 Птн 07:09:10 515631145
>>515628
>В НАЧАЛЕ БЫЛО СЛОВО.
И слово было РЯЯЯЯ!
Аноним 27/09/19 Птн 07:10:14 515632146
Аноним 27/09/19 Птн 07:13:02 515633147
>>515632
А в стиме чета нет.
Аноним 27/09/19 Птн 07:16:19 515634148
>>515633
>А в стиме чета нет.
Чойта?
Аноним 27/09/19 Птн 08:08:10 515638149
>>515634
А, бля, там кароч Эпик им заплатил чтобы те издавались у них эксклюзивно.
Аноним 27/09/19 Птн 08:09:39 515639150
>>515638
Так игруля просто шикарная, сейчас все на ней сидят. Брат вроде жив, тоже гамает, зависимость есть.
Аноним 27/09/19 Птн 15:42:39 515760151
Стикер (191Кб, 500x500)
500x500
>>515302
>Как будет такое же прилагательное про Землю? Земельный? Земляной? Землианский?
Земной
Аноним 27/09/19 Птн 15:53:12 515763152
Я тут делаю "Инновационный" газовый котел для воздушного отопления с аж двумя теплообменниками, долго рассказывать.

В общем, там установка био-метановая с очисткой метана до уверенно выше 90%

Установка за сутки выдает 3 куба чистого метана, там будет принудиловка...

Мне нужно сжигать метан с максимальным КПД без давления, сколько кубов воздуха смешивать на 3 куба метана,

И как лучше смешивать, там будет условно воздуховод в 1.5 метра, там можно производить смешение а дальше горелка, или лучше гарелку чисто под метан а воздух загонять сбоку?

Вроде как в ракетных двигателях максимально замешивают в смесителе...
Аноним 27/09/19 Птн 15:54:46 515766153
>>515763
>гарелка

Горелка

комфорка
Аноним 27/09/19 Птн 16:06:20 515773154
>>515763
>Вроде как в ракетных двигателях максимально замешивают в смесителе...
Вейт, а разве в ракетных двигателях его вообще замешивают перед сжиганием? Оно разве не при первом же контакте с окислителем самовоспламеняется?
Аноним 27/09/19 Птн 17:45:05 515798155
>>515763
Если смешать а потом поджечь - то будет взрыв, как в ДВС и вакуумных бомбах.
Если нужно горение с высоким КПД - то воздух направлять на пламя, как в ракетных печах.
Аноним 27/09/19 Птн 18:26:48 515813156
>>515798
>то будет взрыв


А детонационную печь можно замутить?
Аноним 27/09/19 Птн 18:36:44 515815157
Изображение-663.jpg (149Кб, 800x494)
800x494
Если марсоебам так важно сохранение жизни "Планета в порядке - это людям пиздец", почему при каждом запуске не цепляется какая-нибудь килограммовая капсула с десятикилограммовым движком, с самыми живучими организмами и не отправляется нахуй из солнечной системы. Понятно что на запуске на Марс бабло гораздо лучше пилится. Но именно мой план работоспособный или все же с орбиты не так просто улететь?

Селедка с подвеской, напомнила мне про передачу, которую я смотрел про метеоритоискателей. Тупо гугланул металлоискатели, и ахуел, что низ начинается чуть ли не со ста баксов. Стоит ли пытаться вкатиться в искателя, не для наживы ради, а просто как хобби. В радиусе 30 км от моей деревни примерно такой рельеф как на пике, то ли ледниками так по пидорасило, то ли хз еще от чего.
Такой рельеф скорее плюс или минус к поискам? По ощущениям вроде как более легкие и мелкие частицы должны слетать с этих горочек и на верхушках много чего интересного должно быть или все же равнины лучше в этих делах?
Аноним 27/09/19 Птн 18:40:37 515817158
>>515815
Шанс попасть такими запусками в какую нибудь звездную систему - 0%.
Можно пролететь всю галактику насквозь в каком нибудь space engine, и ты никогда не встретишься со звездой.
Аноним 27/09/19 Птн 18:43:46 515818159
>>515601
Будет. Солнце греют не ядерные реакции, а само вещество. Атомы вещества ведь диссипируют в процессе своей жизнедеятельности, образуя вокруг себя поля, которые мы называем всякими научными терминами. Флуктуации этих полей колеблют соседние атомы - получается тепло. Один атом выделяет мало тепла, но если атомов много, то тепло накапливается.
Аноним 27/09/19 Птн 18:48:32 515819160
Аноним 27/09/19 Птн 18:50:54 515821161
>>515818
Нихуя се, атомы щекотят друг друга и тепло появляется. Я думал все проще, атомы разрушаются - выделяется огромное количество энергии, тоже самое и при соединении водорода в гелий, а тут во как.
Аноним 27/09/19 Птн 20:09:16 515861162
>>515773
>Оно разве не при первом же контакте с окислителем самовоспламеняется?
это справделиво только для гиперголических пар
ндмг + ат
остальное надо поджигать
Аноним 27/09/19 Птн 20:09:36 515862163
Аноним 27/09/19 Птн 20:35:04 515878164
>>515815
жизни людей блядь
бактерии и так выживут пока солнце не поглотит землю
Аноним 27/09/19 Птн 21:14:26 515916165
>>515798
>Если смешать а потом поджечь - то будет взрыв, как в ДВС и вакуумных бомбах.



Надо мутить котел на микро взрывах! вроде как энергетика там на порядок выше!

Аноним 27/09/19 Птн 21:24:25 515926166
>>515821
Ядерные реакции бурно происходют при формировании Солнца, а когда оно сформировано, то ядро представляет собой кучу тяжелых элементов, которым уже некуда дальше объединяться и ядерные реакции происходят по минимуму.

Считаем сколько примерно мощности выделяет один протон. Мощность Солнца делим на массу Солнца и множим на массу протона.
3.828e26 / 1.9885e30 * 1.673e-27 = 3.2206406839326e-31 Вт или 2.0101490359649e-12 эВ/с
Как видим, величина достаточно мизерная, то есть протон вполне способен столько выделять своими флуктуациями.
Аноним 27/09/19 Птн 21:31:40 515938167
>>515926
Выделил тебе за щеку.
Аноним 27/09/19 Птн 23:58:35 516036168
>>514000 (OP)

Что там за макромолекулы нашли на Энцеладе?Это чё??Вроде там и вода есть
Аноним 28/09/19 Суб 00:03:00 516040169
>>515306

Алекса Филипенко забыл
Аноним 28/09/19 Суб 03:17:45 516090170
>>516036
>Что там за макромолекулы нашли на Энцеладе

Органические соединения. Как простые типа метана, этана, ацетилен так и более сложные - метанол, этанол и пр.
Аноним 28/09/19 Суб 08:16:55 516116171
>>514042
унылое зрелище
Невозможно сосредоточится
У хохлов там ничего не менялось с 1980х
Такой монотонный бубнёж даже для одарённых ботаников едва на грани приемлемости

Для обычных и тупых людей необходима компьютерная графика - мультипликация и приятная дикция
Аноним 28/09/19 Суб 08:44:52 516118172
n1mcut.gif (20Кб, 350x350)
350x350
N-117101414.jpg (293Кб, 442x595)
442x595
Аноны, читаю историю советской лунной програмы, нашел пик. Вопрос: что за круглая херня на конце поздних версий Н-1 ?! Это такой обтекатель?! нихуя не пойму.
Аноним 28/09/19 Суб 09:02:40 516119173
>>516116
>Невозможно сосредоточится
У тебя СДВГ?
Аноним 28/09/19 Суб 09:05:17 516121174
>>516118
Православный купол.
Аноним 28/09/19 Суб 09:14:44 516127175
Аноним 28/09/19 Суб 10:35:55 516143176
image.png (113Кб, 3136x159)
3136x159
>>515926
>Считаем сколько примерно мощности выделяет один протон
Чебля
>Мощность Солнца делим на массу Солнца и множим на массу протона.
ЧЕБЛЯДЬ
У тебя ВСЁ СОЛНЦЕ что ли участвует в реакциях, ебанат?
Не пори дебильную математику, за тебя все давно посчитано.
Аноним 28/09/19 Суб 10:36:59 516144177
>>516119
У меня НДМГ, но да, вместо бубнежа лучше книгу почитать. А если делаете познавательный материал, так хоть картинки рисуйте.

видео не смотрел и не собираюсь
Баба страшная(( Аноним 28/09/19 Суб 12:47:19 516162178
Поставьте негра хоть на аву, да хоть кого-нибудь, умоляю!
Сил нет на страхолюдину смотреть
Аноним 28/09/19 Суб 12:52:58 516163179
>>516118
Как обычно в рунете информации сильно меньше, а астронавтикс нихуя не работает.
Это скорей всего ДЛБ, Долговременная Лунная База, но инфа 80%.
Алсо, на пикче у тебя Н-1М, у которой криогеника сверху используется.
Аноним 28/09/19 Суб 12:53:15 516164180
>>516162
НСФВ нажми и все.
Аноним 28/09/19 Суб 13:00:46 516167181
>>516164
это какая-то куклоприблуда? Помню была комбинация клавиш, чтобы заблюрить пикчи, напомни что-ли. хотя она вроде всё скрывает
Аноним 28/09/19 Суб 13:02:20 516168182
>>515815
Без геологического навыка, ты не отличишь метеорит от горной породы, лол
Аноним 28/09/19 Суб 13:13:49 516170183
>>516168
Всмысле не отличу? 80% того что долетает или металлические или с нихуевым содержанием металла. Любой черный (оплавленный/обожженный) камень это метеорит.
Аноним 28/09/19 Суб 13:19:58 516172184
>>516170
откровенная чернота смоется после пары дождей, покроется мхом, грязью. Есть черные базальты и в природе например, или кто-то в костре камень подержит и ты такой "Метеорит!"
Аноним 28/09/19 Суб 13:23:51 516174185
>>516172
https://www.youtube.com/watch?v=EZg4059i1fE
Первый видос по запросу "поиск метеоритов с металлоискателем"
1.20 проверка магнитом, открой любой видос по теме, там будет про магнит. Ну если, конечно, в говне моченые не наебывают обычных людей.
Аноним 28/09/19 Суб 13:54:10 516193186
>>516163
Да она, меня именно интересовала довольно интересная форма головной части. Явно что то грузовое, САС нет. А ссылку на источник что, это может быть ДЛБ не кинешь?! хотелось поподробней почитать.
Аноним 28/09/19 Суб 14:10:04 516207187
>>516174
Так на видео он говорит о том, чтобы искать свежий метеорит под эпицентром взрыва, куда осколки падают. То есть это нечто с конкретной позицией, известным составом, недавнее. А просто в рандомном лесу без наводки ты хуй чо найдешь. Будешь там по 100 раз наклоняться на каждый гвоздь, гайку, крышку из под пива.
Аноним 28/09/19 Суб 14:28:27 516216188
>>514042
Охуенные лекции. Когда работал репетитором по физике, смотрел у него что забыл со школы.
Аноним 28/09/19 Суб 15:33:13 516241189
>>516118>>516163>>516193
Первая пикча - выдумка Вэйда.
На фотографии справа - конструктивно-подобная модель РКН Н1-Л3 в масштабе 1:10, слева - КМП блока В РН Н1 без хвостового отсека с комплексом Л3 без головного обтекателя в масштабе 1:5.
Аноним 28/09/19 Суб 15:51:08 516247190
Аноним 28/09/19 Суб 16:18:52 516261191
Patomskycrater.jpg (199Кб, 940x626)
940x626
1b280fefcfd9bc6[...].jpg (309Кб, 1024x768)
1024x768
Как думаешь, Патомский кратер - космического происхождения? Или это что-то иное?
Аноним 28/09/19 Суб 16:45:55 516276192
>>516261
Нет конечно, он же просто кричит о своём вулканическом происхождени.
Аноним 28/09/19 Суб 16:47:30 516277193
>>516276
Так нет, не нашли следов магмы и трубки магматической.
Так как возраст кратера имеет принципиальное значение, в 2008 году В. И. Ворониным (СИФИБР СО РАН) был предпринят массовый отбор спилов наиболее высоковозрастных лиственниц в нескольких метрах от внешней осыпи кратера и на самом кратере. В результате дендрохронологического анализа обосновано заключение, что около 500 лет назад в процессе формирования насыпного конуса и активной подвижки грунта произошёл массовый вывал деревьев и появилось новое поколение лиственниц с возрастом 400—480 лет.
Аноним 28/09/19 Суб 16:50:56 516278194
>>516277
А это и не обязательно, ёба. Блядь, да он даже не похож на импактные кратеры, ну ты чё?
>Геолого-геохимические исследования в составе комплексных экспедиций 2006, 2008 и 2010 годах свидетельствуют об эндогенных причинах образования кратера, которые, очевидно, связаны с развитием глубинного магматического процесса.
>Установлено, что эруптивный материал в виде глыб песчаников и сланцев среди известняков, выведенный на современную поверхность, был интенсивно карбонатизирован. В результате, в терригенных породах образовалась минеральная ассоциация карбонатных минералов: кальцит, сидерит, анкерит, флюорит. Обнаружение в пределах кратера отдельных глыб терригенных пород с аномальными геохимическими характеристиками даёт возможность предполагать существование на глубине субвулканического магматического тела, определившего вещественные особенности пород Патомского кратера.
>Предполагаемая ранее метеоритная гипотеза происхождения Патомского кратера не подтверждается проведёнными комплексными исследованиями, его образование связано с эндогенными процессами, главную роль в которых играло поступление глубинного потока газовых и флюидных компонентов.
Аноним 28/09/19 Суб 16:51:17 516279195
>>516277
да исключается метеорит в любом случае. Хули он такой конус образовал? от метеорита была бы просто чаша, которую заполнила бы вода, деревья и было бы болотце
Может просто пожар был в тайге, вот тебе и новое поколение лиственниц.
Аноним 28/09/19 Суб 16:55:14 516281196
>>516279
>да исключается метеорит
Геофизики георадарами нашли какое-то металлическое тело под землей, что-то там есть.
Аноним 28/09/19 Суб 17:01:51 516284197
dobycha-rudy.jpg (467Кб, 900x600)
900x600
>>516281
ШОК В ЗЕМНОЙ КОРЕ ОБНАРУЖЕН МЕТАЛЛ
Анон, просто пойди и погугли как выглядят ударные кратеры, и сразу поймёшь что пытаться натянуть метеорит на Патомский кратер очень глупая затея.
Аноним 28/09/19 Суб 17:01:59 516285198
>>516281
естественные минералы какенибудь,или типа металлический ксенолит в глотке застрял.
Аноним 28/09/19 Суб 17:04:58 516287199
>>516284
>ШОК В ЗЕМНОЙ КОРЕ ОБНАРУЖЕН МЕТАЛЛ

Не просто металл, а некий компактный металлический продолговатый предмет.
Аноним 28/09/19 Суб 17:09:24 516289200
>>516287
Про металлоносные пласты породы не слышал никогда?
Аноним 28/09/19 Суб 17:12:17 516293201
>>516289
Геофизики, в отличие от тебя не могут объяснить причины появления этого металлического тела.
Аноним 28/09/19 Суб 17:19:25 516295202
>>516293
Это хорошо, правда в отличие от тебя никто из них высосать метеорит из нихуя не пытается, потому что умеют отличать ударные кратеры от других.
Ты серьёзно думаешь, что столкнувшееся с Землёй на ебанистической скорости тело зарывается в землю и лежит там одним куском, насыпав над собой аккуратный конус породы, при этом ещё и магическим образом не оставлет следов ударного расплава?
Аноним 28/09/19 Суб 18:57:08 516316203
1.png (7Кб, 1239x657)
1239x657
2.png (7Кб, 1239x657)
1239x657
gifs22.gif (6775Кб, 342x465)
342x465
Как выглядит график радиус-время ядра в момент коллапса в нейтронную звезду? Давление вещества постепенно гасит импульс или происходит серия упругих вибраций или по-другому вообще?
Аноним 28/09/19 Суб 19:59:43 516334204
>>516316
Может зависит от массы звезды, температуры, от наличия ассиметричностей во время коллапса?
Аноним 28/09/19 Суб 20:07:34 516337205
>>514000 (OP)
Можно ли гравитационным манёвром у Юпитера направить спутник и увеличенной скоростью прямо в ад нахуй к Солнцу?
Аноним 28/09/19 Суб 20:14:01 516338206
Аноним 28/09/19 Суб 21:39:16 516371207
image.png (14Кб, 705x91)
705x91
>>516167
Наверх посмотри.
Аноним 28/09/19 Суб 21:41:01 516372208
>>516241
Я тоже тебя благодарю, анон.

Заодно извиняюсь за дезу про ДЛБ.
Аноним 28/09/19 Суб 21:42:27 516374209
>>516337
Можно. Но это не очень рационально.
Во первых, аппарат придется делать под слишком большой диапазон температур и энергии, поскольку излучение Солнца у Юпитера в ~25 раз слабее, чем у Земли.
Во вторых, отправка к Юпитеру очень затратна. По топливу гораздо выгоднее сделать несколько маневров подряд у Венеры и Земли.
Да и по времени будет не сильно хуже, так как один пролет к Юпитеру будет длится гораздо дольше, чем даже несколько пролетов Венеры. Скажем. упомянутый этим >>516338 Улисс стартовал в октябре 1990, а над полюсом с максимальной широтой прошел в сентябре 1994, то есть потратил около 4 лет. Parker Solar Probe за те же 4 года совершит 5 гравиманевров у Венеры из семи запланированных.
Аноним 28/09/19 Суб 22:22:54 516392210
Аноним 28/09/19 Суб 23:38:12 516409211
>>516143
Так я не про ядерные реакции написал, а про кое-что другое, о чем ты помыслить не в состоянии.
Аноним 29/09/19 Вск 07:57:44 516662212
>>516409
И нахуя ты про кое-что другое пишешь, если энергия солнца от ядерных реакций, шизик?
Аноним 29/09/19 Вск 07:58:30 516663213
>>516374
Бредишь, как раз от дальних планет наиболее выгодно по топливу упасть на солнце. По времени - да, невыгодно.
Аноним 29/09/19 Вск 08:38:11 516670214
>>516663
>Бредишь, как раз от дальних планет наиболее выгодно по топливу упасть на солнце.
Нет. До Венеры нужно меньше топлива, а наматывать маневры у нее и Земли можно сколько угодно. У того же Паркера запланированно аж 7 маневров.
Нужно ~3.35 км/с сверх второй космической, что бы долететь до Юпитера. Что бы начать наматывать круги мимо Венеры достаточно ~0,65 км/с.
Аноним 29/09/19 Вск 11:22:43 516714215
>>516662
>энергия солнца от ядерных реакций
Это не точно, никто же в солнышко не нырял и не проверял что у нее внутри происходит. То есть это теория, а не 100% факт. Так что вполне позволительно сочинять другие гипотезы и теории.
Аноним 29/09/19 Вск 12:30:22 516741216
>>516714
>это теория, а не 100% факт
Готовь ебальник
Аноним 29/09/19 Вск 13:31:20 516766217
>>516714
>Это не точно
А от чего же тогда на Солнце берется энергия?
Аноним 29/09/19 Вск 16:58:17 516852218
>>516766
В XIX веке вообще думали, что там уголь горит.
Аноним 29/09/19 Вск 17:05:14 516854219
>>516852
Где нибудь можно почитать истории открытий и представлений о космосе? Концентрированно, а не по 20 статьям бегать.
Аноним 29/09/19 Вск 17:43:35 516867220
>>516854
Энциклопедия для детей по астрономии от издательства "Аванта+", 2013 год.
На слова "для детей" не гляди, писали серьёзные астрономы на отличном научно-популярном уровне, вполне сойдёт и для взрослых.
Аноним 29/09/19 Вск 18:03:52 516879221
>>516714
>То есть это теория, а не 100% факт
О, креационист на моём спейсаче.
Аноним 30/09/19 Пнд 01:14:02 517061222
>>516854
>Где нибудь можно почитать истории открытий и представлений о космосе?
Полуркай на ютубе лекции об истории астрономии, например лекции Язева или Сурдина.
Аноним 30/09/19 Пнд 01:48:51 517074223
Такой вопрос - что будет если рядом будут находиться две двойные черные дыры, излучающие гравитационные волны в фазе друг на друга? Будут ли волны складываться, создавая локальное расширение/сжатие пространства-времени или это все бред?
Аноним 30/09/19 Пнд 03:49:14 517084224
>>517074
>создавая локальное расширение/сжатие пространства-времени
Так гравиволны это и есть расширение/сжатие пространства-времени. Представь это как волны на воде от двух камне. Они будут интерферировать.
Аноним 30/09/19 Пнд 12:39:48 517185225
>>517074
Они больше не объединены.
Аноним 30/09/19 Пнд 14:08:33 517226226
Какова была б температура на Венере, будь она на орбите Марса?
Аноним 30/09/19 Пнд 14:14:11 517230227
>>516714
Точно-неточно, а ты ничего и отдалённо близкого по убедительности взамен не предложишь. И у тебя обывательское представление о слове "теория". По-твоему, это когда кто-то остроумно пёрнул в лужу и все ему поддакивают на веру. Между тем как ситуация прямо противоположна, и теория постоянно подвергается всесторонним проверкам в том или ином виде.
Аноним 30/09/19 Пнд 14:20:02 517233228
>>517226
Из-за парникового эффекта такая же, как и щас. Ну, мож, градусов на 100...200 пониже - один хрен не выжить.
Аноним 30/09/19 Пнд 15:05:54 517257229
>>517233
Но ведь и атмосфера сама послабее была бы (и мож намного). Многие газы скомбинировались бы и осели, например. Углекислый газ затвердел бы в полярных шапках, там.
Аноним 30/09/19 Пнд 16:26:08 517307230
food-1681977640.png (407Кб, 640x605)
640x605
Что случится со свежим яблоком если его оставить на поверхности Луны в середине дня? Сразу же и через два месяца?
Аноним 30/09/19 Пнд 16:29:34 517315231
>>516714
Назови мне хоть один 100% факт.
Аноним 30/09/19 Пнд 16:33:50 517328232
Если с НОО сбросить дерево с комом земли и густой кроной(например очень пушистую ель), то сможет ли его крона с ветками и иголками послужить чем-то вроде атмосферного тормоза, чтоб на землю пизданулась не угольная пыль, а ствол, пусть и обгорелый?
Аноним 30/09/19 Пнд 17:01:03 517352233
Аноним 30/09/19 Пнд 17:02:46 517354234
>>517328
Не сможет, а вот кора и древесина могут служить теплозащитой, тут от толщины ствола зависит.
Аноним 30/09/19 Пнд 17:03:17 517356235
Аноним 30/09/19 Пнд 21:38:26 517487236
>>517257
Хуй ты рассчитаешь, какая там атмосфера была бы. Если температура будет хотя бы как на Земле, то никакой углекислый газ никуда не выпадет. Тут-то он не твердеет. Да и на Марсе он только на полюсах твердый и тает сублимируется летом. Вообще, температура и атмосфера очень сильно взаимосвязаны и просчитать что и как развивалось бы, если бы Венера 4 миллиарда лет была на орбите Марса нереально при нынешних знаниях.
Аноним 30/09/19 Пнд 22:31:36 517528237
Вот это дерьмо будет радиоактивным или это обычные видимые фотоны? Если я увижу такую сверхновую, то нужно ли прикрываясь портфелем бежать к метро, или можно наслаждаться видом и доставать телефон?
https://twitter.com/CaliaDomenico/status/1178748703638740993
Аноним 30/09/19 Пнд 22:58:43 517558238
>>517528
Разумеется, там весь спектр. Но оно не страшнее Солнца, от которого тебя защищает атмосфера.
Аноним 30/09/19 Пнд 23:21:57 517588239
>>517558
Если так, то хорошо. потому что у меня в городе нет метро.
Аноним 30/09/19 Пнд 23:30:52 517596240
>>517528
Хуясе, а я сегодня даже на улице не был.
Аноним 01/10/19 Втр 00:02:59 517640241
>>517588
А тебе Бетельгейзе его и не проложит.
Аноним 01/10/19 Втр 00:32:13 517653242
>>517640
Ну и нахрен она нужна тогда?
Аноним 01/10/19 Втр 01:56:34 517663243
Чем ближе к Солнцу, тем время течёт быстрее, в сравнение с земным?
Аноним 01/10/19 Втр 02:45:07 517667244
>>517663
Если испытывать ускорение (т.е. не крутиться вокруг и не падать свободно), то да.
Аноним 01/10/19 Втр 04:53:55 517679245
Название Стоимость ПН Орбита
Ravn $0,2M 0,1т ССО; 500 км
NEPTUNE NS $0,25M 0,0064т 90°; 310 км
Spyder Orbital $1M 0,009 т НЗО
Vector-R $1.5M 0,066 т НЗО
Vector-RE1 $2M 0,035 т CCО; 800 км
Vector-H ~$3M 0,16 т НЗО
VLM $3M 0,16 т НЗО
Jielong-1 (Smart Dragon-1) $6M 0,2 т ССО; 500 км
Куайчжоу KZ-1 $6M 0,43 т ССО; 500 км
Electron $6,95M 0,1 т ССО; 500 км
Landspace LS-1 $8M 0,4 т ССО; 500 км
Куайчжоу KZ-1A $9M 0,25 т ССО; 500 км
INTREPID-1 $9M 0,376 т ССО; 500 км
CZ-11 $10M 0,435 т ССО; 500 км
Terran-1 $10M 1,25 т НЗО
LauncherOne $12M >0,3т CCО; 500 км
RS1 $12M 0,875т CCО; 500 км
Minotaur I $12,5M 0,335т ССО; 740 км
Shavit 1 $15M 0,35 т 90°; 240X600 км
Firefly Alpha $15M 0,63 т ССО; 500 км
Athena I $17M 0,36 т ССО; 800 км
Днепр $20M 0,8 т ССО; 700 км
Рокот / Бриз-КМ (Федер.) $21M 1,15 т ССО; 700 км
Союз 2-1б (Федер.) $23M 8,35 т 51,8°; 200 км
Minotaur IV $23,6M 1,075т ССО; 700 км
Союз-2-1В/БВ Волга $25M 1,4 т ССО; 835 км
CZ-2C $25M 1,4 т ССО; 600 км
CZ-2D $25M 1,3 т ССО; 600 км
PSLV-CA $29,3M 1,2 т ССО; 630 км
PSLV-XL $31,5M 1,75 т ССО; 630 км
Minotaur-C (Taurus XL) $35M 0,93 т ССО; 700 км
Союз 2-1б (Главкосмос 2018 ) $35M 8,35 т 51,8°; 200 км
Epsilon $38M 0,45 т ССО; 500 км
Рокот / Бриз-КМ (Eurockot) $39M-$45M 1,15 т ССО; 700 км
Vega (2015) $44M 1,5 т ССО; 700 км
Циклон-4М $45M 3,35 т ССО; 700 км
GSLV Mark 2 $45M 2,5 т ГПО dV=1800 м/с
Союз 2-1б / Фрегат (Федер.) $47M 4,6 т ССО; 820 км
Союз 2-1б / Фрегат (Главкосмос 2018 ) $48,5M 4,6 т ССО; 820 км
Союз 2-1а / Фрегат (Starsem) $50M 4,2 т ССО; 820 км
Днепр (Iridium 2G) $51,8M 0,8 т ССО; 700 км
Pegasus XL $56,3M 0,225т ССО; 700 км
Зенит 3SLБ / ДМ SLБ $60M 4,5 т ГПО dV=1800 м/с
CZ-3A $60M 2,6 т ГПО dV=1800 м/с
CZ-4B $60M 2,3 т ССО
CZ-4C $60M 2,9 т ССО
Falcon 9 v1.2 (2016) $62M 5,3 т ГПО dV=1800 м/с
Протон М / Бриз М (Федер.) <$65M 3,7 т ГCО
Протон М / ДМ-03 (Федер.) <$65M 3,0 т ГСО
Протон М / Бриз М (ILS) ~$65 6,5 т ГПО dV=1500 м/с
Falcon 9 v1.2 (Iridium Next, 2016) $67,9M 5,3 т 86.4°; 667 км
CZ-3B $70M 5,1 т ГПО dV=1800 м/с
CZ-3BE ~$70M 5,5 т ГПО dV=1800 м/с
CZ-3C $70M 3,8 т ГПО dV=1800 м/с
Союз СТБ / Фрегат МТ $80M 4,9 т ССО; 820 км
Antares 230 $80-85M 6,7 т 51,6°; 200 км
Зенит 3SL / ДМ SL $80-90M 6,5 т ГПО dV=1500 м/с
Ariane 62 $85M 5,0 т ГПО dV=1500 м/с
Falcon 9 v1.1 (AMOS 6, 2015) $85M 5,5 т ГПО dV=1800 м/с
Vulcan-Centaur 504 (2020) $85M 4,75 т ГПО dV=1800 м/с
Falcon 9 v1.1 (NLS II) $87M 4,85 т ГПО dV=1800 м/с
Falcon Heavy (2016) $90M до 8 т ГПО dV=1800 м/с
Falcon 9 v1.2 (USAF GPS-III, 2017) $96,5M 3,68 т 55°; 20200 км
Falcon 9 v1.1 (USAF 2015) $97M 4,85 т ГПО dV=1800 м/с
Falcon 9 v1.2 (Sentinel-6A, NLS-II, 2017) $97M 1,4 т 66°; 1336 км
H-IIA 202 $98M 4,0 т ГПО dV=1800 м/с
H-IIA 204 $98M 6,0 т ГПО dV=1800 м/с
Atlas V 401 (2017) $109M 4,767т ГПО dV=1800 м/с
Atlas V 411 (2017) $115M 5,964т ГПО dV=1800 м/с
Atlas V 501 (2017) $120M 3,778т ГПО dV=1800 м/с
Atlas V 421 (2017) $123M 6,903т ГПО dV=1800 м/с
Atlas V 431 (2017) $130M 7,715т ГПО dV=1800 м/с
Atlas V 511 (2017) $130M 5,248т ГПО dV=1800 м/с
Ariane 64 $130M 11,5 т ГПО dV=1500 м/с
Atlas V 401 (JPSS-2, NLS-II, 2017) $132,4M 2,93 т CCО; 824 км
Falcon Heavy (2012) $135M 21,2 т ГПО dV=1800 м/с
Atlas V 521 (2017) $135M 6,477т ГПО dV=1800 м/с
Delta II 7320-10С ~$140M 1,6 т ССО; 833 км
Delta II 7920-10С ~$140M 3,0 т ССО; 833 км
Atlas V 531 (2017) $140M 7,448т ГПО dV=1800 м/с
Atlas V 541 (2017) $145M 8,287т ГПО dV=1800 м/с
Atlas V 551 (2017) $153M 8,899т ГПО dV=1800 м/с
Atlas V 401 (Landsat-9, NLS-II, 2017) $153,8M 2,623 т 98,2°; 705 км
Delta IV Medium+(4,2) (2014) $164M 6,39 т ГПО dV=1800 м/с
Falcon Heavy (Air Force 2015) $165M 21,2 т ГПО dV=1800 м/с
H-II B $182M 16,5 т 30,4°; 300 км
Атлас V 551 (USAF STP-3, 2017) $191,1М ПО ГСО
Атлас V 541 (MSL) $195М 4,75 т 11,2 км/с
Атлас V 551 (New Horizons) $213М 0,478т 16,21 км/с
Delta IV M+ (5,2) (2014) ~$225M 5,49 т ГПО dV=1800 м/с
Delta IV Medium+ (5,4) (2014) ~$225M 7,3 т ГПО dV=1800 м/с
Ariane 5 ES $240M 20,0 т 51,6°; 300 км
Ariane 5 ECA $240M 9,5 т ГПО dV=1500 м/с
Атлас V 541 (Mars 2020) $243М 4,75 т 11,2 км/с
Vulcan-Centaur 564 (2020) $260M ГПО dV=1800 м/с
Delta IV Heavy (2015) $389M 14,220т ГПО dV=1800 м/с
Аноним 01/10/19 Втр 09:00:20 517707246
1569909596353.jpg (112Кб, 600x418)
600x418
Почему во вселенной существуют пустоты?
Аноним 01/10/19 Втр 09:20:06 517710247
>>517307
Сразу же ничего не будет.
Через два месяца найдешь сублимированный сухофрукт.
Под пережаренной каменной корочкой будет хуита которую можешь использовать для приготовления компота.
Аноним 01/10/19 Втр 09:22:19 517711248
>>517707
ТАК ВЫШЛО.
Наука не отвечает на вопросы "почему" да "зачему", это предполагает наличие стремления, цели, какого-то замысла.
Замысла нет.
Аноним 01/10/19 Втр 09:26:41 517712249
Аноним 01/10/19 Втр 09:46:38 517714250
>>517712
Но он прав. Поиск Великих Замыслов - это удел людей с религиозным типом мышления.
Аноним 01/10/19 Втр 10:40:16 517720251
>>517714
Каких замыслов?
Слово "почему" не обязательно подразумевает "боженька сделол)))". А причину формирования например, может из-за гравитации войды появились или ещё чего. Я не знаю, поэтому и спрашиваю.
>>517711
> это предполагает наличие стремления, цели,
А познание не цель? Стремление узнать больше о войдах и их формировании - религиозный тип мышления?

Короче, кто о чем, а лысый о расческе. Если вы в таком обычном вопросе видите Бога, разумные замыслы и т.п., то у вас проблемы.
Аноним 01/10/19 Втр 10:41:18 517722252
>>517663
Нет. Время везде одинаково.
Аноним 01/10/19 Втр 10:46:22 517723253
>>517230
>это когда кто-то остроумно пёрнул в лужу
Это называется гипотеза. Теория это когда ты берешь гипотезу и развиваешь ее, добавляешь деталей, матана, подгоняешь под известные эксперименты.
Аноним 01/10/19 Втр 10:48:23 517725254
>>517315
Да хоть три: Земля шарообразна, Солнце светит, эфир существует.
Аноним 01/10/19 Втр 10:53:13 517726255
>>516766
Тепловая энергия берется из окружающего вакуума из-за неравномерностей в процессах диссипации нуклонов и прочего вещества.
Аноним 01/10/19 Втр 11:32:57 517730256
>>517230
Теория может быть хорошо проработана, но быть ошибочной - рано или поздно споткнется на некотором эксперименте.
Аноним 01/10/19 Втр 12:18:27 517735257
>>517725
С первым пунктом ты обосрался, остальные можно не рассматривать. Это если вообще реагировать на твой вброс хоть сколько-нибудь серьёзно.
Аноним 01/10/19 Втр 12:19:59 517736258
>>517723
Почитай определение и не коверкай его такими словами, пожалуйста.
Аноним 01/10/19 Втр 12:20:09 517737259
>>517730
Тем не менее, ничего более достоверного, чем научная теория, человечеству неизвестно.
Аноним 01/10/19 Втр 13:25:20 517764260
>>517707
Следы от квантовых флуктуаций, когда вселенная была пиздюшной, и на нее действовали эффекты квантового мира.
Аноним 01/10/19 Втр 17:03:15 517826261
Аноним 01/10/19 Втр 19:30:37 517885262
если вся материя быыла сгустком меньше атома, то я происхожу от всего? можно сказать что я включаю в себе всю вселенную, или то что вся вселенная включает меня?
Аноним 01/10/19 Втр 19:40:18 517893263
Аноним 01/10/19 Втр 21:07:46 517912264
Аноним 01/10/19 Втр 21:13:25 517914265
Аноним 01/10/19 Втр 21:26:49 517916266
>>517914
Наоборот, конечно, чем глубже в колодец, тем медленнее идет время. Хуй знает, кто отвечает, что быстрее.
Аноним 01/10/19 Втр 21:27:19 517917267
В фильме "К Звездам" сцена с крышкой антенны это отсылка к Касcини?
Аноним 01/10/19 Втр 22:08:55 517920268
Поисните, почему в 100% случаев пиздят о "Вселенной, сжатой в точку" до БВ? Если Вселенная конечная (конечный объем, точнее), то тут ок, вопросов нет. Но если она бесконечна (насколько я знаю, етот вопрос не решен), то как может что-то конечное превратиться в бесконечное? Как же соображения непрерывности и все такое?
Аноним 01/10/19 Втр 22:32:40 517924269
EFfWVXRXUAMZtiB1.jpg (141Кб, 1200x900)
1200x900
Маск реально собрал ракету в чистом поле и она должна там же и стартовать? Всякие сверхчистые цеха заводов не нужны?
Аноним 01/10/19 Втр 22:41:27 517928270
>>517924
Это не ракета, а бак с двигателями для тестов в величину корабля. В космическом корабле должна быть система управления, больше датчиков, электричество для СЖО, радиаторы, солнечные батареи, системы связи и прочие приблуды, которых в этом нету.
Аноним 01/10/19 Втр 22:54:02 517935271
>>517928
роскос, просто соси и не завидуй
Аноним 01/10/19 Втр 23:46:54 517952272
>>517920
У нее конечное количество массы. Объем вон увеличивается постоянно.
Аноним 02/10/19 Срд 00:02:24 517958273
>>517952
>У нее конечное количество массы.
Схуяли? В видимой части конечное, а во всей вообще - с чего ты взял?
>Объем вон увеличивается постоянно.
Эт если он конечный. А если он уже бесконечный, куда ему увеличиваться?
Аноним 02/10/19 Срд 00:29:08 517970274
>>517667
объясни, плз, почему влияет ускорение?
Аноним 02/10/19 Срд 00:44:53 517973275
>>517928
>Это не ракета, а бак с двигателями для тестов в величину корабля.
Бак с двигателями, который летает = ракета.

>В космическом корабле должна быть система управления
Она там есть. И датчики есть, иначе нахуй такие тесты?

>>517924
Да. А ты думал, все ракеты в чистых комнатах всегда собирали? Нет, и Фау-2, и Союзы, и Редстоуны всякие собирали в обычных таких цехах. В чистых комнатах всякие спутники и аппаратуру делают. Ну, и сами двигатели.
Аноним 02/10/19 Срд 00:47:05 517975276
Посмотрел фильм "К звёздам" с Питтом. Разочарован. Не сайфай, не космофентезя, не космоопера и не просто что-то явно космическое, а скорее разговорный фильм и история о человеческих взаимоотношениях в антураже космоса. Если бы история происходило не в будущем "пару дней спустя", а, скажем, лет 200-300 назад и была бы о мореплавателях или также лесоходцах, исследователях гор или пустынь - ничего бы не изменилось. Однако, этот фильм вызвал у меня вопрос: в сцене около Нептуна, который подкрашен таинственного синего цвета с мистическим тусклым свечение, сам газовый гигант нормально так видно. Но как бы мы видели Нептун в реале? Насколько там яркое солнце? Поясните на примере хотя бы лампочки разных ватностей или материалов, чтобы я хотя бы примерно почувстовавал
Аноним 02/10/19 Срд 00:54:08 517980277
>>517975
Фильм не смотрел, но по твоему описанию все нормально. Освещенность в яркий солнечный день на Земле - около 100 000 люкс, без учета атмосферы - 135 000 (https://ru.wikipedia.org/wiki/Освещённость). Нептун в 30 раз дальше, значит освещенность там в 900 раз меньше. То есть около 1500 люкс. По таблице примеров, это как пасмурный день на Земле, на порядки больше того, когда пропадает различение цвета.

Короче, разгадка в том, что глаз может приспосабливаться к освещенности и нормально работать в очень широком диапазоне.
Аноним 02/10/19 Срд 01:00:15 517986278
>>517975
>Насколько там яркое солнце?

Солнечная постоянная на орбите Земли 1360 Вт/м2.
Солнечная постоянная на орбите Нептуна 2 Вт/м2.
Аноним 02/10/19 Срд 01:00:35 517987279
ХЗБ.gif (940Кб, 627x502)
627x502
Если звезда или планета с глючным коллайдером схлопнется в ЧД, заметный глазу аккреционный диск образуется сразу же (из окрестной пыли, например) или понадобится время на сбор вещества?
Аноним 02/10/19 Срд 01:01:45 517989280
>>517987
Науке это неизвестно.
Аноним 02/10/19 Срд 01:05:31 517994281
>>517989
Печяльненько, чо.
Аноним 02/10/19 Срд 01:06:45 517997282
>>517986
>Солнечная постоянная на орбите Нептуна 2 Вт/м2.
Мне это ни о чём не говорит, объясни нормально - лампочками, свечками, спичками хотя бы
Аноним 02/10/19 Срд 01:09:26 518001283
>>517997
Солнце там в 700 раз слабее светит чем на Земле.
Аноним 02/10/19 Срд 01:17:41 518005284
1) Орбиты первых четёрх планет вокруг солнца более лимения круглые и ровные, по крайне мере по сравнению с Плутом уж точно - орбиты друг другу не пересекают, лежат в одной плоскости и всё такое. Но насолько они круглые? Вдруг орбита, скажем Земли такова, что в апогей очень близко подбирается к орбите Марса, а перегей - к орбите Венеры, и что Марс и Венера вращаются ровно по кругу, а Земля - овалом? Земля это пример, вопрос ей не ограничен

2) Как создать наиболее эффективный по охлажденю зонд для вращения на низкой орбите (много ниже орбиты Меркурия), чтобы исследовал солнце, но сам не сгорал настолько долго, насколько это возможно?

3) Как проводить исследования Венеры? Допустим, советский венероход, показавший миру единственные фоточкм голой венерианской писечки, на самом деле просто неудачно сел не туда - в зону кратера супервулкна, просто вулкана, или равнины битуумных гейзеров, и что Венера на самом деле куда более приятная и близкая Земле температурой у поверхности (что конечно, мало вероятно), и тогда стоит другим венероходам сесть в другом месте им будет не так жарко, что, однако, никак не отменяет ебического давления и плотных облаков. Как же тогда епердвать радио или какие они там используют сигнлы наверх, в космос, Земле? Как нам обустроить двач обследовать Венеру?
Аноним 02/10/19 Срд 01:27:59 518008285
>>518005
>Но насолько они круглые?
https://ru.wikipedia.org/wiki/Эксцентриситет_орбиты

>Как создать наиболее эффективный по охлажденю зонд
Дать денег инженерам, чтобы они создали.

>Как же тогда епердвать радио или какие они там используют сигнлы наверх, в космос, Земле?

В каком смысле как?
Аноним 02/10/19 Срд 01:30:19 518009286
>>517986
>Вт/м2
В данном случае (солнечный свет) сойдет, но вообще эта хуйня мало о чем говорит. Будь там хоть мульон ватт на квадрат радиоволн, например, их ты не увидишь.
Аноним 02/10/19 Срд 01:33:34 518010287
>>518009
>но вообще эта хуйня мало о чем говорит. Будь там хоть мульон ватт на квадрат радиоволн

На на видимый свет приходится 46% энергии излучаемой Солнцем.
На инфракрасное изучение 47%.
Ну УФ 7%.
Аноним 02/10/19 Срд 01:34:55 518011288
>>518010
>В данном случае (солнечный свет) сойдет
Аноним 02/10/19 Срд 01:38:11 518012289
>>518011
>В данном случае
А мы о данном случае (Солнце) тут и говорим, а не о гипотетических звездах.
Аноним 02/10/19 Срд 01:44:20 518014290
>>517924
Её из силосной башни прямо на ферме в ракету переделали?
Аноним 02/10/19 Срд 01:56:12 518016291
>>518005
3) Венера вся примерно одинаковой температуры из-за того, что ее плотная атмосфера крайне эффективно переносит тепло. Такая уж у нее охуительная газовая оболочка, что там нет ни климатических поясов, ни сезонов, ни даже смены дня и ночи (освещенность тоже не меняется, всегда одинаковый сумрак).

Передавать сигналы будут так же, как передавали их советские зонды — через орбитальный ретранслятор. Прямиком до Земли пробиться сигналу слишком тяжело, да и энергии на это уйдет слишком много.
Аноним 02/10/19 Срд 02:06:50 518017292
>>518016
>Венера вся примерно одинаковой температуры из-за того, что ее плотная атмосфера крайне эффективно переносит тепло.

Наоборот. Она такая горячая потому, что ее атмосфера крайне эффективно не пропускает тепло. Луркай "Парниковый эффект".
Аноним 02/10/19 Срд 02:09:37 518018293
>>518017
Про конвекцию и скорость ветра на Венере слышал?
Аноним 02/10/19 Срд 02:11:17 518019294
>>518018
>Про конвекцию и скорость ветра на Венере слышал?
И как скорость ветра не Венере противоречит сказанному мною?
Аноним 02/10/19 Срд 02:13:18 518020295
>>518019
Никак. Только вот ты стал писать про "наоборот" в ответ на пост, который расписывал одинаковость температуры.
Аноним 02/10/19 Срд 02:17:10 518021296
>>518020
>Только вот ты стал писать про "наоборот"
Но ведь я все верно написал. Тот анон ошибся утверждая, что "атмосфера Венеры крайне эффективно переносит тепло". Она украине плохо его переносит, потому что почти целиком состоит из углекислого газа, поэтому там так жарко.
Аноним 02/10/19 Срд 02:19:29 518022297
>>518021
И как парниковость газа влияет на его способность переносить тепло?
Аноним 02/10/19 Срд 02:23:13 518023298
j4ssshki66411.jpg (32Кб, 318x469)
318x469
185207350.jpg (118Кб, 600x399)
600x399
jupiter-and-red[...].jpg (51Кб, 610x410)
610x410
>>517973
>Фау-2, и Союзы, и Редстоуны всякие собирали в обычных таких цехах.

Но хотя бы в цехах, а не на открытом воздухе же.
Аноним 02/10/19 Срд 02:24:03 518024299
>>518022
>И как парниковость газа влияет на его способность переносить тепло?
Почитай статью в вики о парниковом эффекте.
Аноним 02/10/19 Срд 02:26:51 518025300
Аноним 02/10/19 Срд 03:28:34 518028301
>>518021
Подумай, через что лучше передается тепло, через вакуум или через газ? Через разреженный газ или через плотный? Через плотный газ или через твердое тело? Простейшие физические принципы говорят нам, что чем больше давление газа, тем выше его теплопроводность, поэтому передача тепла на Венере идет в десятки раз более эффективно, чем на Земле.

Под толщей атмосферы у самой поверхности все температурные различия уже сглажены, и различий в температуре реально почти нет.


>>518024
Это такой троллинг тупостью? Парниковый эффект не имеет абсолютно никакого отношения к теплопроводности, и вызывается он исключительно разницей в прозрачности атмосферы для солнечного спектра и для лучей инфракрасного диапазона.

Видимый свет (с длинами волн, соответствующих температуре Солнца) легко проникает глубоко внутрь атмосферы сквозь прозрачный для него углекислый газ и поглощается поверхностью, но обратно излучается спектр с температурой уже не 5800К, а всего 700К, т.е. гораздо более длинные волны. Для них толща углекислого газа все равно что глухая стена, и просто так улететь в космос они уже не могут, вместо этого постоянно поглощаясь и переизлучаясь молекулами газа и поднимая его температуру.
Аноним 02/10/19 Срд 03:46:43 518030302
>>518028
>Подумай, через что лучше передается тепло, через вакуум или через газ?

А какая связь между тем, что в вакууме нет передачи тепла при помощи теплопроводности и конвекции и тем фактом, что на Венере высокая температура из-за парникового эффекта вызывного тем, что основу атмосферы Венеры составляет парниковый газ СО2.
Аноним 02/10/19 Срд 03:52:05 518032303
>>518028
>обратно излучается спектр с температурой уже не 5800К, а всего 700К, т.е. гораздо более длинные волны. Для них толща углекислого газа все равно что глухая стена, и просто так улететь в космос они уже не могут

Следовательно вот это утверждение неверно

>плотная атмосфера крайне эффективно переносит тепло
Аноним 02/10/19 Срд 03:56:04 518034304
>>518030
Никакой, на это и указывали. Высокая температура от парникового газа и равномерная температура от высокой плотности не связаны вообще никак.

Если бы на Венере была атмосфера не из углекислого газа, а из, например, прозрачного для инфракрасных волн азота, эффективная передача тепла бы все равно была, как и вызванное ей равенство температуры по всей планете, а парникового эффекта бы не было; я вообще хуй знает, зачем ты его приплел, когда мы говорили только про теплопроводность.
Аноним 02/10/19 Срд 03:57:43 518035305
>>518032
Это натуральный нон секвитур, ты просто хуйню несешь уже. Не вижу смысла дальше участвовать в этом троллинге тупостью.
Аноним 02/10/19 Срд 03:58:27 518036306
>>518034
>Никакой, на это и указывали

Указывали на другое, а именно на то, что атмосфера Венеры плохо пропускает тепло.
Аноним 02/10/19 Срд 06:28:11 518042307
>>518034
>Если бы на Венере была атмосфера не из углекислого газа
Начались виляния жопой, кхех.
Аноним 02/10/19 Срд 06:54:56 518045308
>>517920
Если посмотреть на плоскость с торца, она будет выглядеть линией. Линия с торца выглядит точкой. Точка не имеет размера. Это тебе пример из математики, как бесконечное может оказаться конечным.
Аноним 02/10/19 Срд 07:00:21 518046309
>>518045
>Если посмотреть на плоскость с торца
Невозможно если плоскость бесконечная. :3
Аноним 02/10/19 Срд 07:01:58 518047310
>>518032
Ты слышал когда-нибудь фразу одно другому не мешает?
Речь шла не о том, почему на Венере тепло, а о том, почему на Венере равномерно тепло.
Аноним 02/10/19 Срд 07:03:28 518048311
>>518046
Вот, пространство вокруг тебя бесконечно. Твое существование в нем невозможно?
Аноним 02/10/19 Срд 07:08:29 518050312
>>518048
>Вот, пространство вокруг тебя бесконечно
Так.

>Твое существование в нем невозможно?
Не вижу никакой логической связи с твоим первым утверждением.

Ты лучше расскажи как собрался смотреть на бесконечную плоть с торца.
Аноним 02/10/19 Срд 07:09:02 518051313
>>518050
>на бесконечную плоскость с торца.
fix
Аноним 02/10/19 Срд 08:04:21 518057314
>>518005
>1) Орбиты
Гугли орбиты планет, куча есть картинок и анимаций.

>2) Как создать наиболее эффективный по охлажденю зонд
Паркер.

>3) Как проводить исследования Венеры? Допустим
Мы знаем, что Венера вся такая, для этого не нужно садиться в разных местах. Алсо, это не венероход был, потому что он никуда не ходил, это был просто лендер. А так, сейчас много есть концептов исследования Венеры, ищи в новостных тредах, венеротреде, АМС треде, там постили.
Аноним 02/10/19 Срд 08:56:39 518061315
Почему Макс делает свой новый корабль на улице? Ещё и варит стыки сваркой. Это ещё что за пиздец?
Аноним 02/10/19 Срд 09:08:31 518069316
>>518061
Потому что может. Зачем всё переусложнять?
Аноним 02/10/19 Срд 09:22:16 518073317
>>518069
Оно у него вообще полетит?
Аноним 02/10/19 Срд 10:12:21 518082318
Из чего состоят чёрные дыры? Пиздос.
Аноним 02/10/19 Срд 11:19:55 518093319
>>518082
Вопрос "из чего состоят" неявно подразумевает, что объект описывается квантовополевой теорией с иерархией масштабов. Для черных дыр это не так, для их описания на "микроскопическом уровне" (скажем, из которого можно получить статистическое описание их термодинамики) требуется гипотетическая квантовая теория гравитации, которая пока не известна. Известно, что это не будет квантовая теория поля (по крайней мере, с конечным количеством видов полей).
Аноним 02/10/19 Срд 11:25:42 518094320
>>518073
Через пару месяцев сам увидишь. Его более мелкий Стархоппер уже летал на одном таком двигателе, теперь пришла очередь башни на трех движках.
Аноним 02/10/19 Срд 11:33:19 518095321
Когда люди исследуют гравитацию и научатся ей управлять это будет прорыв покруче колеса или двигателя.
Аноним 02/10/19 Срд 11:37:53 518096322
Максимально тупые вопросы ITT.

1) Как так вышло, что белые люди проебали б-жественное наследие фон Брауна и ушли с головой в Спейс Шатл, которые угандошил дохуя огурцов и оказался дорогущей хуйней без задач, да еще и оставил токсичное говно "SLS" после себя? Можно ли сказать, что совочек спасло от такого же движения нахуй закрытие Бурана? Может ли Маск повторить эту порочную траекторию со Старшипом, если не будут реализованы цели по его многоразовости/орбитальной дозаправке/ретропропульсивной посадке?

2) Можно ли отснять Венеру в метровом разрешении на низкоорбитальный SAR, сохранить охулиарды байт данных на массив из накопителей и отправить его на Землящку в спускаемом аппарате с SEP, вместо того, чтобы ебаться с передачей этой инфы по радио? Где подводные камни? Большая масса возвращаемого модуля? Недостаточная защита накопителей от радиации?
Аноним 02/10/19 Срд 11:48:32 518098323
>>518096
>шатол
поначалу идея казалось хорошей, а когда поняли что не очень - было поздно что-то менять. плюс изначально шатол планировался в комплексе с многочисленными орбитальными станциями денег не дали, в такой ситуации без него хуево было бы
>совок
учитывая что у них хватило мозгов бездумно скопировать шатол - это просто случайность что в ссср/рф удалось сохранить пилотируемые полеты
>муск
хуй знает. общая концепция на бумаге хороша, но по мере разработки возникает много проблем. от способа решения этих проблем зависит будет ли старшип говном или нет
>венера
размеры и вес аппарата прежде всего, радар в том числе. ну и защита от радиации тоже
но прежде всего это не слишком целесообразно сейчас
в теории можно конечно
Аноним 02/10/19 Срд 12:04:06 518100324
>>518098
> это просто случайность что в ссср/рф удалось сохранить пилотируемые полеты
Деньги на Буран удачно закончились до того момента, как успели выпилить Союзы.
>>518096
> Можно ли отснять Венеру в метровом разрешении на низкоорбитальный SAR, сохранить охулиарды байт данных на массив из накопителей и отправить его на Землящку в спускаемом аппарате с SEP, вместо того, чтобы ебаться с передачей этой инфы по радио? Где подводные камни? Большая масса возвращаемого модуля? Недостаточная защита накопителей от радиации?
Полагаю, передать это по радио будет все же быстрее и дешевле, чем ебаться с возвращаемыми аппаратами.
Аноним 02/10/19 Срд 12:28:29 518105325
>>518100
>Полагаю
В пдфке по EnVision по передаче всё совсем печально.
Аноним 02/10/19 Срд 14:23:25 518169326
>>518046
Ну на полуплоскость, например, можно, а она тоже бесконечная. Но я тут один хуй не вижу связи с вопросом о "соображениях непрерывности". Если функция имеет конечное значение при t < t0, а при t = t0 - бесконечное, то это разрыв. При этом нарушаются все законы сохранения, например.

Вот, кстати, что нашел в педивикии:
>The Big Bang was "small": It is misleading to visualize the Big Bang by comparing its size to everyday objects. When the size of the universe at Big Bang is described, it refers to the size of the observable universe, and not the entire universe.[129]

То есть, в принципе, это не противоречит ситуации со Вселенной бесконечного объема.

Кстати, в той статье нигде не упоминается размер сингулярности (конечный или бесконечный): говорится только о плотности вещества, которая стремится к бесконечности в точке сингулярности.
Аноним 02/10/19 Срд 15:41:58 518215327
>>518100
>Деньги на Буран удачно закончились до того момента, как успели выпилить Союзы.
Буран и не предполагался в качестве основного корабля, им должен был стать ТКС. Из одного из них модуль "Заря" потом сделали.
Аноним 02/10/19 Срд 15:44:30 518216328
>>518215
> основного корабля, им должен был стать ТКС.
А зарю зачем тогда разрабатывали?
ТКС в пилотируемом варианте был чисто для военных, и стоил столько дохуя что никакой заменой союзу быть не мог.
Аноним 02/10/19 Срд 15:51:10 518217329
>>518215
хуйню несешь
ТКС дебильная концепция, хуй знает зачем каждый раз возить с собой модуль станции
К тому же модуль называется ФГБ, и из него практически все станции салют и мир
>>518216
Потому что ТКС говно
Аноним 02/10/19 Срд 16:01:17 518221330
>>518096
>1) Как так вышло, что белые люди проебали б-жественное наследие фон Брауна и ушли с головой в Спейс Шатл, которые угандошил дохуя огурцов
>и оказался дорогущей хуйней без задач
Ключевое слово тут - оказался. При чем во многом по чисто организационно-политическим причинам. Шаттл это часть программы STS, а она в свою очередь инженерный проект "дороги фон-Брауна". Суть: делаем шаттлами орбитальную станцию, орбитальную заправку, пилим орбитальные буксиры, все выводим шаттлами. За эту концепцию так или иначе держались вплоть до конца шаттла - и концепция оказалась провальной.

Однако главный секрет шаттла это то что он не шаттл те челнок. Это короткоживущая орбитальная станция. Тогда правда его цена перестает быть огромной и становится вполне разумной на фоне полученных результатов.
> Можно ли сказать, что совочек спасло от такого же движения нахуй закрытие Бурана
Нельзя. Буран изначально придумывался исходя из того что американцы всё врут на тему своего STS (не может солдат столько брюквы сьесть). Буран изначально делался сверхдорогой иобой без задач, из расчета что когда наша разведка узнает секрет шаттла - вот тогда задачи и появятся. То есть если американцы обосрались, советы просто насрали себе в штаны потому что американцы обосрались, а советы решили что американцы обосрались специально и это хитрый план.
Аноним 02/10/19 Срд 16:06:12 518223331
>>518221
Это как ты решил что концепция провальная если сначала на станции вообще денег не давали, а дали только на фридом/МКС через лет 25, в которой не было и нет никакой орбитальной инфраструктуры вообще?
Аноним 02/10/19 Срд 16:42:25 518234332
>>518223
>Это как ты решил что концепция провальная если сначала на станции вообще денег не давали, а дали только на фридом/МКС через лет 25, в которой не было и нет никакой орбитальной инфраструктуры вообще?
Решил не я и решил результатам МКС и шаттла
а) сделать челнок, то есть постоянно многоразово дешево на орбиту грузовик не удалось. По выводам комиссии Орбитер так и остался опасным экспериментальным аппаратом с неустранимыми проблемами.
б) сборка чего-то на орбите оказалась адом, и сейчас главный результат МКС это "избегать орбитальной сборки когда возможно".
Пример: сколько времени роисские космонавты делали задачу "пара фотак и соскоб в месте дырки на союзе"? Пару часов? А остальные задачи ВКД - это же смех один! То что на земле потные мексы делают на несколько минут - в космосе это сначала годами подготовка на земле, потом часы работы с травмами: Как я понял, значительная часть микротравм приходится на кисть и пальцы из-за того что приходится продавливать пальцами скафандр под давлением - в результате за время долгих ВКД особо удачливые успевают стереть кожу до мяса, заработать гематомы на пальцах и добиться отслоения ногтей.
Аноним 02/10/19 Срд 16:46:36 518235333
>>518223
> в которой не было и нет никакой орбитальной инфраструктуры вообще
И кстати ее там по этому и нет. МКС то здоровая, хотя бы экспериментально орбитальную инфраструктуру там могли сделать. Но делать не стали, и планировать не планируют.
Аноним 02/10/19 Срд 16:57:15 518237334
>>518234
>а)
зачем ты выдаешь проблемы реализации за неверность концепции?
>б)
сможешь привести список что на МКС специально сделано для работы вне станции? А на скафандре? Ниче там нет кроме поручний и шлюза, и на скафандрах аналогично нет никаких конструктивных особенностей для работы
Так зачем ты опять выдаешь фактически отсутствие внятной реализации на ошибочность концепции?
Аноним 02/10/19 Срд 17:08:08 518240335
Аноним 02/10/19 Срд 17:49:03 518245336
>>518237
>>а)
>>б)
>зачем ты выдаешь проблемы реализации за неверность концепции?
Потому что эту концепцию предлагают для реализации в реальном мире а не в воображении любителей фентази. Подразумевается что концепция так придумана, что осуществима в реальном мире сейчас. За разумные сроки, с разумными технологиями и разумными деньгами.

Если это не так - это неверная концепция.
> для работы вне станции? А на скафандре? Ниче там нет кроме поручний и шлюза, и на скафандрах аналогично нет никаких конструктивных особенностей для работы

Факты: С момента первого выхода в космос прошло более полувека (54 года). За это время было придумано и проведено множество экспериментов в области орбитальной сборки. Придумано и сделано куча инструментов, методик для работ в космосе в невесомости - при чем придумано умнейшими людьми планеты. Тем не менее то что на земле занимает минуты, в космосе занимает часы, приносит травмы, несет риск для жизни и требует многогодовой подготовке к конкретной миссии на земле.

> Ниче там нет
Ну то есть результат полувековой работы человечества по ВКД - ниче там нет. Пиздец.

Мой тезис - если полвека пытались и получился хуй, значит это блять сложна. А концепции которые считают что это проста - неверные и идут нахуй.

Твой тезис какой? ЗОГ не дает привинтить на МКС волшебные вундервафли превращающий ВКД с орбитальной сборкой в легкую прогулку?
Аноним 02/10/19 Срд 18:34:46 518249337
>>515798
>взрыв, как в ДВС
Маня, иди падмойса, в ДВС именно что горение смеси. Детонация (именно взрыв) убивает двигло к хуям.
Аноним 02/10/19 Срд 19:33:41 518261338
>>518245
>Потому что эту концепцию предлагают для реализации в реальном мире а не в воображении любителей фентази. Подразумевается что концепция так придумана, что осуществима в реальном мире сейчас. За разумные сроки, с разумными технологиями и разумными деньгами.
ну давай расскажи что там за технологические ограничения не позволяют сделать дешевый и безопасный шатол
>За это время было придумано и проведено множество экспериментов в области орбитальной сборки.
ссылки на эксперименты
>Придумано и сделано куча инструментов
летающее кресло с шаттла и космический шуруповерт?
>Тем не менее то что на земле занимает минуты, в космосе занимает часы, приносит травмы, несет риск для жизни и требует многогодовой подготовке к конкретной миссии на земле.
и все это связано с банальным неудобством, ну кроме рисков
по решению проблем удобства особых попыток не было
а раз не было попыток - нельзя говорить о провальности концепции
про подготовку вообще пиздец, зачем ты выдаешь общую подготовку за подготовку к конретной ВКД?
>Ну то есть результат полувековой работы человечества по ВКД - ниче там нет. Пиздец.
Работы ВКД начались только с шаттлов в 90х, хаббл, мкс и прочее. И кстати нормально там все делалось, цеплялись на канадарм и охуенно было, опора есть, не надо лазить по поручням и все такое.
>полвека пытались
Еще раз - никто не пытался 50 лет наладить орбитальную сборку, все что было и есть - это выходы на небольшой ремонт

>Твой тезис какой?
Такой что орбитальная сборка сама по себе подразумевает что собираем мы что-то большое, дорогое и сложное. А так как на это нет денег - не особо целесообразно практиковать орбитальную сборку. Почитай про концепт сборки JIMO например
Аноним 02/10/19 Срд 19:50:23 518264339
>>518050
Блядь, ну пусть это будет полуплоскость. Это принципиально?
Аноним 02/10/19 Срд 20:09:00 518268340
> ну давай расскажи что там за технологические ограничения не позволяют сделать дешевый и безопасный шатол
С чего ты взял что обязательно технологические? Чувак, мы в реальности а не в твоем воображении - если ограничения не технологические это по прежнему невозможность.
>ссылки на эксперименты
> по решению проблем удобства особых попыток не было
> а раз не было попыток - нельзя говорить о провальности концепции
> вы все врети
Пофиксил, не благодари.

> летающее кресло с шаттла и космический шуруповерт?
Что тебя смущает? Что нет волшебных единорогов, а раз нет, значит нещитово?

> Еще раз - никто не пытался 50 лет наладить орбитальную сборку, все что было и есть - это выходы на небольшой ремонт
Мы не можем сделать что-то простое, это оказалось сложно. Почему? Потому что мы не пробовали делать что-то очень большое и сложное! Очень большое и сложное обязательно получится, в отличие от простого.
Логика уровня бог.

> Такой что орбитальная сборка сама по себе подразумевает что собираем мы что-то большое, дорогое и сложное.
Нет. Есть фундаментальный принцип от простого к сложному. Ты его отрицаешь?
Сначала воткнуть кабель и закрутить гайку, потом сборка JIMO, потом сборка из мелких простых элементов всей конструкции. Если на уровне "воткнуть кабель" у нас пиздец, значит концепция не работает.
Аноним 02/10/19 Срд 20:30:59 518275341
>>518268
>нет ответа про заявленные тобой ограничения
>нет ответа про заявленные тобой эксперименты
>нет ответа про заявленные тобой особые инструменты

>Логика уровня бог.
>Если на уровне "воткнуть кабель" у нас пиздец, значит концепция не работает.
твой пример это примерно если мне надо починить кран в ванной, я чето там руками покрутил, неудобно было пиздец, руки натер как на ВКД, но вроде работает. а все потому что у меня нет разводного ключа
Понел теперь?

Я кстати тоже могу орать что вывод грузов целиком это тупиковый путь, ну там ограничения по диаметру, солнечне панели и радиаторы не влезут а вручную раскрывать же западло, так допиздимся что космонавтика вообще не нужна а может ты к этому и идешь так как являешься МКС-хейтером
Аноним 02/10/19 Срд 20:49:42 518276342
Что такое звуковая волна и чем она принципиально отличается от радиоволны? Почему скорость радиоволны в воздухе примерно = скорости света, а скорость звука примерно 300 м/с? Разве это не чит передавать звук на огромные расстояния радиоволнами?
Аноним 02/10/19 Срд 20:51:48 518277343
>>518276
звук - колебания вещества
радиоволна - поток фотонов
Аноним 02/10/19 Срд 21:35:03 518284344
>>518276
Звук распространяется в какой-то среде (материи), находящейся в пространстве. Частицы среды (молекулы газов воздуха, скажем) то кучкуются погуще, то разлетаются поширше. Если рассмотреть какую-то выделенную точку, то в ней будет колебание плотности среды. Как волна, это колебание распространяется во все стороны, с угасанием. Отсюда, кстати, следует, что звуковая волна не может быть произвольной мощности: отрицательной плотности не бывает, да и для огромной положительной банально среды вокруг не хватит.

Свет распространяется в светоносном эфире электромагнитном поле. Грубо говоря, в каждой точке пространства есть несколько величин, характеризующие электрические и магнитные силы, действующие на тела в этой точке. Световая волна, она же фотон, в любой момент находится в одной точке (хотя квантмех загадочная щтука, и все точки получаются "размытыми"), распространяется по прямой, а не во все стороны. Колебания заключаются в том, что прирост тех самых величин поля (в точке, где фотон вот прямо щас) дрыгает туда-сюда.

Как-то так вроде.
Аноним 02/10/19 Срд 22:14:08 518287345
>>518234
>Как я понял, значительная часть микротравм приходится на кисть и пальцы из-за того что приходится продавливать пальцами скафандр под давлением - в результате за время долгих ВКД особо удачливые успевают стереть кожу до мяса, заработать гематомы на пальцах и добиться отслоения ногтей.
Ты заебал постить свою же пасту так, как будто это заявление какого-то авторитетного человека.
Аноним 02/10/19 Срд 22:26:39 518292346
1475845234382.png (229Кб, 825x1340)
825x1340
>>518284
>Свет распространяется в электромагнитном поле
Нет. Свет порождает электромагнитное поле. Фотон это переносчик электромагнитного взаимодействия подобно тому, как звуковая волна - переносчик звукового давления. И вот это "облако" виртуальных фотонов, скачущих между электрическими зарядами, и образует поле.

А вот в какой среде распространяется свет - сие тайна великая есть. Считается, что "просто так", а светоносного эфира не существует. Его действительно не существует, если подходить к вопросу в ньютоновском смысле, считая, что эфир это такой газ.
Но у нас есть т.н. физический вакуум, который нихуя не пустой, умеет куда-то растягиваться, имеет конечную скорость передачи взаимодействий и вообще ведёт себя как-то не как пустота. И здесь следует поставить ребром вопрос об элементарности элементарных частиц.

Вот адроны и лептоны это материя. Из них можно слепить материальную среду и гонять по ней звук либо фазовые состояния, создавая бегучие и стоячие волны. Фонон, магнон, ротон и.т.п. это квазичастицы, из которых состоит "постматерия". Они сами по себе никак не влияют на количество элементарных частиц, существуя как бы "поверх" них, но ведут себя подобно элементарным частицам.

Тогда логично предположить, что если существует постматерия, то может существовать и предматерия. Такое нечто, для которого наши элементарные частицы являются квазичастицами. Брана из М-теории описывает как раз нечто подобное. И вот если светоносный эфир существует, то сделан он из предматерии. Т.е. на нашем "этаже" он нефизичен, но порождения его физичны.
Аноним 02/10/19 Срд 23:05:53 518295347
>>518264
Половина бесконечной плоскости тоже какбы бесконечна.
>>518276
>Разве это не чит передавать звук на огромные расстояния радиоволнами?
Товарищ Маркони, поздравляю с подключением.
Аноним 02/10/19 Срд 23:13:20 518297348
>>518275
>>нет ответа про заявленные тобой ограничения
>>нет ответа про заявленные тобой эксперименты
>>нет ответа про заявленные тобой особые инструменты
Ты не обосновал ни одного своего утверждения ничем кроме как неявного "я верю". Почему я тебе должен чтото там постить? При этом ты споришь с очевидными вещами - человеки в космосе с ВКД 54 года, но нужны ДОКАЗАТЕЛЬСТВА что люди учились делать орбитальные сборки. Пиздец.


https://en.wikipedia.org/wiki/EASE/ACCESS

https://www.nasa.gov/centers/johnson/pdf/584726main_Wings-ch3e-pgs130-156.pdf
Куча указаний на разработанные методы и инструменты. Да, волшебных единорогов нет.

NASA ran a series of tests beginning with a deployable solar power wing experiment on Discovery’s first flight
(Space Transportation System [STS]-41D in 1984) to validate the construction techniques that would be
used to build the ISS.

These shuttle crew members were trained extensively for their respective missions

>твой пример это примерно если мне надо починить кран в ванной, я чето там руками покрутил, неудобно было пиздец, руки натер как на ВКД, но вроде работает. а все потому что у меня нет разводного ключа

Оставаясь в пределах твоей аналогии: разводной ключ пока не только не изобрели, но даже непонятно что это такое, в попытках изобрести две суперждержавы потратили полвека и триллионы долларов и непонятно возможен ли этот "разводной ключ" на современном техническом уровне. Все что изобрели за полвека и триллионы долларов - это завинчивать руками.
И тут ты такой - нужно потратить еще триллионы долларов и десяток лет времени точно всё будет зуб даю.

Это и есть пиздец, концепция не работает.

>Я кстати тоже могу орать что вывод грузов целиком это тупиковый путь, ну там ограничения по диаметру, солнечне панели и радиаторы не влезут а вручную раскрывать же западло
Я раз что ты не поехал крышей совсем как тут некоторые.

Идея в том что в коридоре "вывод грузов целиком" тупик дальше. Основную коммерческую и научную деятельность ведут аппараты целиком собранные на земле.

> так допиздимся что космонавтика вообще не нужна
Пилотируемая космонавтика вот для этого собственно и нужна. Что бы поехавшим "ряяя счас настроим цилиндров онейла" тыкать прямо носом в то что ничего мы не можем. И для того что бы не пропустить момент когда сможем.
Аноним 02/10/19 Срд 23:14:45 518298349
>>518287
>Ты заебал постить свою же пасту так, как будто это заявление какого-то авторитетного человека.
Маня, это не не моя паста. Это цитата анона которую я превратил в пасту. Вот ему стыдно наверное хахаха. Там еще ссылка на твиттор с доказательствами. Ее я пощу когда очередной дебил орет ряяяяя ниверю.
Аноним 03/10/19 Чтв 00:27:11 518309350
>>518297
Бляяяя
>ссылки
ты их хоть читал сам? это как раз успешные проекты
причем их совсем немного
а помнишь как ты пару постов назад задвигал про огромное количество экспериментов и инструментов? дай угадаю, ты пиздаболил даже не глядя в гугл?
>в попытках изобрести две суперждержавы потратили полвека и триллионы долларов
ясно, то есть ты посчитал абсолютно все расходы на пилотируемую космонавтику как затраты на исследования орбитальной сборки
но вот незадача, сам ты привел лишь пару экспериментов и универсальный канадарм, который не только для ВКД годится
>Идея в том что в коридоре "вывод грузов целиком" тупик дальше
нет, он уже достигнут практически - 10м диаметр на слс и 7-9 на старшипе
>Что бы поехавшим "ряяя счас настроим цилиндров онейла" тыкать прямо носом в то что ничего мы не можем.
А что надо сделать чтобы смочь? Наверно нарочито искажать факты против ВКД-работ и топить за исключительно целые аппараты?
А может все таки стоит дальше развивать ВКД, ну например поставить пару канадармов чтобы покрывать всю МКС, что-нибудь придумать с перчатками скафандров?
Аноним 03/10/19 Чтв 05:14:06 518361351
>>518295
>Товарищ Маркони, поздравляю с подключением.

Я может ошибаюсь, но разве господин Маркони первый запилил радиотелефонию? Он же вроде телеграфом передал, ключом?
Аноним 03/10/19 Чтв 05:50:31 518366352
Может ли так быть, что жизнь в космосе существует в молекулярных облаках в виде полициклических ароматических углеводородов?
Аноним 03/10/19 Чтв 06:34:42 518367353
Все спят что ли?
Аноним 03/10/19 Чтв 06:35:18 518368354
Пол-седьмого уже, пора просыпаться.
Аноним 03/10/19 Чтв 06:36:29 518369355
Аноним 03/10/19 Чтв 06:42:23 518370356
>>518295
> Половина бесконечной плоскости тоже какбы бесконечна.
Торец-то есть, есть куда смотреть.
>>518045
Аноним 03/10/19 Чтв 06:44:34 518371357
>>518370
>Торец-то есть, есть куда смотреть.
Как может быть торец у бесконечной плоскости? Ты идешь его ищешь, а его нет, ведь она бесконечна.
Аноним 03/10/19 Чтв 07:09:13 518373358
>>518277
Какой размер фотона у радиоволн низкой частоты?
Аноним 03/10/19 Чтв 08:30:30 518375359
>>518371
У бесконечной ленты есть целых джва торца
Аноним 03/10/19 Чтв 08:31:59 518376360
>>518375
>У бесконечной есть
Если что-то где-то заканчивается, значит оно не бесконечно.
Аноним 03/10/19 Чтв 15:25:17 518418361
>>518309
> ты их хоть читал сам?
Я то да. А вот ты ни их ни чего другого не читал, я внезапно понял.
И именно по этому требовал от меня ссылок.

> ряяяя пара экспериментов пиздаболил немного незадача

Я кажется понял в чем твоя проблема. Ты считаешь что орбитальная сборка это такая волшебная эльфийская хрень для отработки которой нужны волшебные обряды. И по этому нет этой сборки - обряды не проведены, злые силы мешают.

Задам простой вопрос: вот первый выход в открытый космос это отработка орбитальной сборки? По твоему "хахаха конечно нет. это же не волшебный обряд N2". В реальности же орбитальная сборка состоит на 99% из простых операций "выйти из шлюха", "добраться до места проведения работ", "сунуть болт в отверстие" и так далее. Безо всякого волшебного обряда N2 типа "сварка в вакууме" или волшебного "ох какое волшебство завинтить болт" которые занимают 1%.

И жопа в том что эти базовые примитивные операции так и остались адом. Опасными для жизни (не шутка), рутинно травмирующими экипаж, пиздец стрессом, очень дорогими по баблу, требующие годов подготовки на земле(ссылку я привел), и атски потребляющими время. И все ради того что бы закрутить пару гаек.

То есть все эти ужасы и миллиарды что бы решить проблему решаемую за 15 минут на земле мексом за постабаксов.

> стоит дальше развивать ВКД
Стоит конечно развивать, что за вопрос. И развивают. Но надеятся и малафьей заливать, как ты, уже нет. Всё, капут. Концепция оказалась неудачной. Удачная концепция хуйнуть готовый хабитат и посещать его по мере необходимости. А когда ресурс кончится либо затопить либо спустить вниз старшипом.

А вот когда будет совсем другая концепция и технологии - например телеуправляемые роботы, тогда может быть. И под будет имеется в виду что БУДЕТ. То есть робот возьмет болт и сунет его куда надо как мекс на земле, за пару секунд. А не "мы примотали к федору скотчем дрель ряяя".

> ну например поставить пару канадармов чтобы покрывать всю МКС
И ты считаешь что это уберет тот ад которым является орбитальная сборка?
И ты считаешь что НАСА это такие дебилы которые за ~35 лет не догадались?

>, что-нибудь придумать с перчатками скафандров?
Проблема стоит полвека. Она была ясна сразу после первого ВКД.
За полвека пока чего-то ничего не придумали.
И тут ты такой "ряяя счас все порешаем"
Не порешаешь.

>искажать факты против ВКД-работ
Чувак, ты уже прокололся что по вопросу ничего не читал кроме фантастики. Ты даже pdf по ссылке не читал.

> нет, он уже достигнут практически - 10м диаметр на слс и 7-9 на старшипе
Более узкие диаметры осваивали полвека. Ну вот 9 метров старшипа тоже будут полвека.
А еще есть надкалиберные обтекатели, 18 метров муска, и главное - надувные хабитаты.

Еще раз, ты рассуждаешь с позиций фентази. "Диаметр +100 метров иначе несчитово, нет розовых единорогов". Я рассуждаю с позиций реализма - плюс 5метров это пиздец дохуя.

> ты посчитал абсолютно все расходы на пилотируемую космонавтику как затраты на исследования орбитальной сборки
Не все, а например половину. Или даже треть. Каждый ВКД это исследование орбитальной сборки - рутинное повторение экспериментов 60-х.
Аноним 03/10/19 Чтв 16:04:19 518429362
>>518376
Множество натуральных чисел тоже заканчивается "в одну сторону".
Аноним 03/10/19 Чтв 16:14:44 518437363
screenshot2018-[...].png (1186Кб, 1414x744)
1414x744
skylabsunshades.jpg (36Кб, 620x216)
620x216
>>518261
>Работы ВКД начались только с шаттлов в 90х, хаббл, мкс и прочее.
Ващет работы во время ВКД начались еще на Skylab в семидесятых. Станцию неоднократно чинили снаружи, включая установку двух тепловых экранов.
Аноним 03/10/19 Чтв 16:24:36 518440364
Аноним 03/10/19 Чтв 16:26:07 518441365
>>518437
>Ващет работы во время ВКД начались еще на Skylab в семидесятых. Станцию неоднократно чинили снаружи, включая установку двух тепловых экранов.
Я же говорил что он ничего про ВКД не знает же. Ну вот - еще одно доказательство.
Аноним 03/10/19 Чтв 16:32:00 518442366
Пусть два фотона одной частоты одновременно выпущены из одной точки в одном направлении, с одинаковой поляризацией, но строго в противофазе. Как я понимаю, интерференция должна быть такая, что их какбе и нет вообще. Можно ли такую ситуацию отличить от того, как если бы их вообще не было? Или такого быть не могет?
Аноним 03/10/19 Чтв 16:37:04 518444367
>>518309
> 10м диаметр на слс и 7-9 на старшипе

Кстати:

https://en.wikipedia.org/wiki/BA_2100

BA_2100 Pressurised volume: 2250 m3
ISS Pressurised volume: 915 m3

Для производственных нужд хватит с головой.
Аноним 03/10/19 Чтв 16:44:52 518446368
>>518440
Ты не понимаешь природу интерференции фотонов. Фотон это не волна, а частица. Поэтому интерференция у них это не то же самое что интерференция волн на воде. Как летели два фотона, так и будут лететь, а если ебанут о атом то возбудят его.
Аноним 03/10/19 Чтв 16:55:39 518454369
Что это всё значит? Можно просто объяснить совсем для тупых (меня)? https://youtu.be/xejyeBOHArE
Аноним 03/10/19 Чтв 17:07:00 518463370
Читали науку интерстеллар Кипа Торна?
Аноним 03/10/19 Чтв 17:22:38 518466371
>>518454
Вот тут пиздато проясняется: https://habr.com/ru/post/468381/

Если кратко, то это скорее доказательство многомировой интерпретации квантмеха, нежели отправка сообщений в прошлое (хотя здесь тонкая грань, ибо многомировая интерпретация сама по себе подразумевает возможность путешествий в прошлое не совсем тривиальным образом).

Т.е. частицы могут иметь несколько интерференций одновременно, которые могут работать вразжопицу, чем херят всевозможное теории скрытых параметров. Здесь мы получили на экране один горб из частиц, якобы потерявших свою волновую функцию. Но если извлечь данные из вспомогательных электронов - обнаружится, что горб на самом деле это две интерференции, наложившиеся друг на друга и скрывающие друг друга. Сам факт прочтения этих данных меняет результат эксперимента - мы видим скрытую интерференцию, которая на самом деле произошла.

В многомировой интерпретации всех этих парадоксов нет. Мы сначала неявно-отсекаем одну группу параллельных реальностей (сохранив в кванте информацию о том, что именно отсекли), затем обратно склеиваем, затем отсекаем другую группу и получаем обособленную реальность. Но когда мы извлекаем из кванта информацию - обнаруживается, что это не обособленная реальность, а две, у которых интерференция перекрывает друг друга.
Аноним 03/10/19 Чтв 18:03:45 518477372
>>518466
Уф...Спасибо, стало гораздо понятней. А как тогда быть с квантовы бессмертием? Возможно ли оно?
Аноним 03/10/19 Чтв 18:38:28 518483373
Аноним 03/10/19 Чтв 19:16:38 518488374
>>518217
>ТКС говно
А не спеть ли м пойти ли тебе нахуй?
ТКС охуенен и опередил время.
Если бы не кококогептилофобия была бы охуенная машина до сих пор.
Аноним 03/10/19 Чтв 19:18:14 518490375
>>518488
ну давай расскажи нахуй он нужен если ты не собираешь станцию мир на каждом полете
Аноним 03/10/19 Чтв 20:43:44 518506376
>>518371
Написал же, полуплоскость.
>>518376
Что насчёт луча скажешь?
Аноним 03/10/19 Чтв 21:22:52 518513377
Какой уровень радиации в космосе? А то я чет гуглю и нет никакой информации.
Аноним 03/10/19 Чтв 21:56:43 518521378
>>518513
Где именно в космосе? Он довольно большой.
Аноним 03/10/19 Чтв 22:52:39 518540379
>>518521
За орбитой луны и до Марса.
Аноним 03/10/19 Чтв 23:21:12 518549380
>>518540
Из https://en.wikipedia.org/wiki/Health_threat_from_cosmic_rays
>Astronauts on Apollo and Skylab missions received on average 1.2 mSv/day and 1.4 mSv/day respectively.[20]
>Estimates are that humans unshielded in interplanetary space would receive annually roughly 400 to 900 mSv (compared to 2.4 mSv on Earth) and that a Mars mission (12 months in flight and 18 months on Mars) might expose shielded astronauts to roughly 500 to 1000 mSv.[28]
Короче, в районе 1-3 миллизивертов в сутки обещают. Хз правда, что насчет солнечных вспышек.
Аноним 03/10/19 Чтв 23:23:27 518550381
Аноним 04/10/19 Птн 00:24:24 518553382
D1ZaMW0AEqruF.j[...].jpg (226Кб, 2048x1152)
2048x1152
Что это за хуёвины?
Аноним 04/10/19 Птн 00:39:00 518557383
Почитал немного про ракету Pluto, у которой рабочее тело- забортный воздух, и появились некоторые мысли. Что, если сделать орбитальный самолёт с подобной системой движения? Что-то вроде шаттла, только с воздухозаборником на брюхе, с большими баками для воздуха внутри корпуса и силовой установкой, способной нагревать этот воздух для достижения нужной тяги. Выводится этот аппарат с помощью РН на орбиту, поработал там- рабочее тело заканчивается. Тогда орбитальный самолёт делает нырок в атмосферу( что-то вроде нырка шаттла, которым пугали КПСС), наполняет баки воздухом и снова выходит на орбиту. Цикл повторяется.
Если вышеперечисленное верно, то можно создать целую систему электромагнитных ускорителей/ малых ракет, которые выводили бы грузы за атмосферу, но не на орбиту, а данный аппарат "подхватывал" бы их и выводил бы на орбиту. Помимо этого таким аппаратом можно было бы ловить космический мусор и инспектировать/корректировать орбиты спутников.
Аноним 04/10/19 Птн 00:51:20 518558384
>>518513
>Какой уровень радиации в космосе?

Разный.
Аноним 04/10/19 Птн 00:59:58 518559385
>>518557
А неплохая идея. На Земле вряд ли получится обеспечить требуемую холодопроизводительность для сжижения азота/кислорода, но на Марсе и Венере углекислый газ можно сжижить просто сжатием хотя тоже охлаждать придется.
04/10/19 Птн 01:04:43 518560386
>>518557
эта хуйня работает примерно до 5-8 махов. для орбиты нужно 25.
Аноним 04/10/19 Птн 01:12:16 518561387
>>518557
>наполняет баки воздухом и снова выходит на орбиту

А за счет чего он снова должен выйти на орбиту? Для этого ему придется запустить свой ядерный двигатель и засрать атмосферу радиацией.
Аноним 04/10/19 Птн 01:12:50 518562388
>>518558
Насколько разный? При вспышках какой будет уровень?
Аноним 04/10/19 Птн 01:45:42 518564389
>>518562
>Насколько разный?
Если ты на низкой орбите вокруг Земли, под зашитой ее магнитного поля, как МКС, то за год ты получишь дозу около 200 миллизиверт. Если ты поднимешься чуть выше и попадешь в радиационные пояса, то там уже будут десятки миллизиверт в ЧАС. Выйдя за пределы радиационных поясов будет примерно в 2 раза выше, чем на МКС.

Если говорить о Солнечной системе цвелом, то чем ближе ты к Солнцу, тем выше плотность потока электромагнитного излучения и заряженных частиц. Чем дальше от Солнца тем ниже плотность потока электромагнитного излучения и заряженных частиц от Солнца, но вы плотность потока заряженных частиц приходящих из внешних источников.


Аноним 04/10/19 Птн 07:57:53 518596390
>>518442
Энергия-то никуда не девается, анон.
Аноним 04/10/19 Птн 07:58:46 518598391
>>518490
Но я собираю станцию мир на каждом полете. Проблемы?
Аноним 04/10/19 Птн 08:00:21 518600392
>>518596
Куда девается энергия единичных фотонов, когда они годами путешествуют между звёзд, и испытывают красное смещение при расширении вселенной в то время, как они преодолевают расстояние от звезды до наблюдателя? Поскольку в пути длина их волны увеличивается, а значит, уменьшается энергия.
Аноним 04/10/19 Птн 08:06:36 518602393
>>518600
В гравитацию, вот куда. Летит такой фотон и воздействует на черную дыру в центре галактики.
Аноним 04/10/19 Птн 08:50:26 518613394
spacesuit.jpg (108Кб, 625x484)
625x484
>>518287
Запоздало, но вдруг кому нибудь понадобится.
Вот, например, целая работа этому посвящена
https://www.ncbi.nlm.nih.gov/pubmed/20922881
>A significant number of astronauts sustain hand injuries during extravehicular activity training and operations. These hand injuries have been known to cause fingernail delamination (onycholysis) that requires medical intervention. This study investigated correlations between the anthropometrics of the hand and susceptibility to injury.
Пик - табличка из статьи. В среднем ~9,5% риск травмы руки при ВКД.
Аноним 04/10/19 Птн 09:50:26 518619395
Аноним 04/10/19 Птн 10:45:57 518635396
>>518560
Это в низких слоях атмосферы. В разряженной атмосфере и тем более в вакууме тяга должна быть больше
Аноним 04/10/19 Птн 10:56:15 518636397
>>518635
это как больше если рабочего тела все меньше?
Аноним 04/10/19 Птн 11:25:04 518642398
>>518636
Так и сопротивление атмосферы меньше, а рабочее тело будет "зачерпываться" в более плотных слоях.
Аноним 04/10/19 Птн 12:24:42 518646399
>>518600
Просто исчезает в никуда, ни в какую потенциальную энергию она не уходит.

Закон сохранения энергии это ж не настоящий закон, а просто следствие симметрии физических процессов относительно обращения времени. Если обратить время и получить сокращающуюся вселенную, то фотоны будут приобретать энергию из ниоткуда.
Аноним 04/10/19 Птн 13:04:47 518652400
Аноним 04/10/19 Птн 13:20:27 518654401
>>518600
Рассеивается в эфире.
>>518602
Вот этот в первом предложении верно говорит. Энергия уходит в общее гравитационное поле. Ведь напряженность гравитационного поля это по сути температура эфира. Фотон стареет, рассеивает вокруг свою энергию - окружающий эфир чутка греется - общее гравитационное поле подпитывается. Потом когда-нибудь рассеянная этим фотоном энергия будет участвовать в приталкивании вещественных тел друг к другу.
Аноним 04/10/19 Птн 13:39:20 518657402
>>518652
А закон сохранения энергии работает для процесса расширения вселенной?
Аноним 04/10/19 Птн 13:43:57 518658403
>>518654
>Энергия уходит
Мозги у тебя походу уже ушли.
Аноним 04/10/19 Птн 15:01:39 518673404
>>518652
Если прочитаешь эту самую статью, то выяснишь, что это не фундаментальный закон, а просто следствие теоремы Нётер (симметрии времени), которая работает только локально и в плоском пространстве.

Если можешь в английский, то неплохо расписано в FAQ в https://www.reddit.com/r/askscience/wiki/astronomy — пункты:
>Universe expansion - Does the expanding universe conserve energy?
>Universe expansion - Why doesn't the redshifting of photons by expansion violate the conservation of energy?
Аноним 04/10/19 Птн 15:48:37 518692405
>>518673
>Закон сохранения энергии не фундаментальный закон
Ну все, теперь тебе осталось только написать в Нобелевский комитет и премия у тебя в руках
Аноним 04/10/19 Птн 16:25:17 518706406
>>518682
>>518692
А вот и закономерные подрывы спелеоматериалистических пидорашек подъехали, нойс.
Аноним 04/10/19 Птн 16:52:16 518715407
>>518477
>Уф...Спасибо, стало гораздо понятней. А как тогда быть с квантовы бессмертием? Возможно ли оно?
Возможно, разумеется. Однако, экспериментально проверить его не-субъективно - ОЧЕНЬ нетривиальная задача.

Мы можем взять, например, выборку больных с какими-то тяжёлыми пороками сердца, которым впоследствии сделали пересадку сердца или клапана. До пересадки они могли откинуть копыта от любого стресса, а потом всё стало норм. И мы обнаружим, что до пересадки в жизни им несказанно везло, а после в их жизни стало больше неудач, ведь как только от их груди убрали "квантовый пистолет" - они стали способны пронаблюдать эти неудачи. Однако для постороннего наблюдателя всё это - не более, чем ошибка выжившего.

Можно залезть в большой ящик с тысячей человек и атомной бомбой и тыкать там на кнопку, ловя осечки. Так можно было бы пронаблюдать групповое квантовое бессмертие, но внешним людям ты тем самым не докажешь ничего.

По-настоящему годные эксперименты в этой области начнутся тогда, когда появятся методы редактирования памяти индивида. В этом случае можно будет обойтись без убийств и самоубийств.
Аноним 04/10/19 Птн 17:56:42 518726408
Почему частица становится в супер позицию?
Аноним 04/10/19 Птн 18:00:32 518728409
На каком этапе сейчас ЯРД? Что и кем сейчас разрабатывается и на каком оно этапе? В чем основные сложности ядерки, что до сих пор нет ничего практичного?
Аноним 04/10/19 Птн 18:10:14 518729410
>>518728
на этапе "в принципе сделать можно, но решение сложностей нецелесообразно"
разработок нет уже лет 30, недавно НАСА заявила о интересе
сложности в материалах турбины которые выдерживали бы нужные температуры и общем отводе тепла
нет ничего практичного потому что сами по себе они не имеют смысла, нужны только как часть программ по серьезному освоению космоса, а на это денег не дают после лунных полетов
Аноним 04/10/19 Птн 21:22:33 518761411
Аноним 04/10/19 Птн 21:33:17 518767412
>>518761
И что ты будешь по этому поводу деять?
Аноним 04/10/19 Птн 21:40:12 518772413
>>518767
Смотреть@охуевать
Аноним 04/10/19 Птн 22:05:32 518780414
>>518729
То есть выход — осваивать космос на химии до тех пор, пока он не станет достаточно важным для политиков, чтобы начать финансировать ядерного буксира?
Аноним 04/10/19 Птн 22:08:04 518781415
>>518780
Ну, можешь основать глобальную корпорацию по производству носков, заработать миллиарды, а потом их вложить в разработку ЯРД.
Аноним 04/10/19 Птн 22:24:32 518791416
>>518780
ага ага опять политики мешают
но обосновать экономическую целесообразность с цифрами ты конечно же не сможешь, только невнятные крики про науку и прогресс
Аноним 04/10/19 Птн 22:33:16 518796417
>>518791
А какая экономическая целесообразность проёбанного на F-35 триллиона долларов? Неужели дополнительные 10 миллиардов из бездонного военного бюджета на буксир ради НОУКИ это так много? Может НОУКА и ПРОГРЕСС не нужны? Что сказать то хотел?
Аноним 04/10/19 Птн 22:39:42 518799418
>>518796
Повышение обороноспособности страны конечно же.
10 миллиардов кстати хватит только на телескоп от производителя Ф-35 лол
>на буксир ради НОУКИ
Какой НОУКИ? Что ты будешь возить на нем, компоненты лунной базы, мощные межпланетные зонды? Но они тоже денег стоят, причем дохуя, и этих денег как раз нет. А нет денег на такие проекты - нахуй нужен буксир если на нем нечего возить?
Аноним 04/10/19 Птн 23:27:18 518823419
>>518799
10 миллиардов в год, алё. За 10 лет это почти стоимость Апполона и вполне реальный срок для разработки пихла. Причём оборонный бюджет даже не почувствует. На разработку лунной базы можно направить остальной бюджет насы с СЛС и аналогичного бесполезного.
Аноним 04/10/19 Птн 23:28:07 518824420
>>518823
Да и 10 миллионов в год на одно пихло — это оверкилл. Явно меньше денег потребуется.
Аноним 04/10/19 Птн 23:30:22 518826421
>>518824
>Явно
Бля, вот как можно заранее знать, сколько тебе потребуется, если нет технологии, материалов, и так далее? Ладно построить небоскреб - все технологии есть, материалы известные. Сложить, умножить, посчитать. А тут как?
Аноним 04/10/19 Птн 23:32:42 518829422
Аноним 05/10/19 Суб 00:06:43 518832423
>>518799
>Повышение обороноспособности страны конечно же.
За триллион баксов можно было бы 33 тысячи (!!!) F-15E наклепать и задоминировать к хуям собачьим всю планету. О какой обороноспособности идёт речь вообще? Вся эта программа - один сплошной военно-политический и экономический саботаж.
Аноним 05/10/19 Суб 03:06:21 518874424
>>518832
>3 тысячи (!!!) F-15E наклепать и задоминировать к хуям собачьим всю планету.
Где ты столько летчиков возьмешь? Что бы подготовить одного летчика нужно несколько миллионов долларов.
Аноним 05/10/19 Суб 03:27:53 518876425
>>518874
>Где ты столько летчиков возьмешь?
На сдачу с этой сверхгигантской суммы. Стоимость лётчика это в любом случае 5-10% от стоимость самолёта, не больше.
Аноним 05/10/19 Суб 03:35:14 518878426
Аноним 05/10/19 Суб 03:39:21 518880427
>>518878
Кислород крайне распространение в космосе, а водород вообще самый распространенный элемент. В космосе полно воды.
Аноним 05/10/19 Суб 07:19:26 518887428
EuropasOceanKPH[...].jpg (599Кб, 1024x768)
1024x768
>>518878
Да не то, что бы сильно дохуя.
Аноним 05/10/19 Суб 09:06:30 518892429
>>518878
От вулканизма. В мантии ее дохуя.
Аноним 05/10/19 Суб 09:07:37 518893430
>>518887
Бля, а ведь по Европе можно будет плавать насквозь, как в звездных войнах по планете Набу.
Аноним 05/10/19 Суб 10:59:33 518905431
Аноним 05/10/19 Суб 11:00:34 518906432
>>518878
Кометы принесли.
Аноним 05/10/19 Суб 14:31:23 518947433
Янепонил, кварковые и преонные звёзды мыслятся спрятанными под горизонтом событий или как? Ведь у нейтронной звезды почти уже предельно минимальный радиус, а у кварковой - на порядок меньше. Значит, это по сути попытка описания структуры чёрной дыры?
Аноним 05/10/19 Суб 14:41:56 518949434
>>518947
>Янепонил, кварковые и преонные звёзды мыслятся спрятанными под горизонтом событий или как?

Видимо да. Но ты пойми, что это лишь гипотезы, продукт размышлений теоретиков, у них задача - придумывать новые модели которые бы не противоречили общей теории относительности.
Аноним 05/10/19 Суб 14:43:57 518950435
>>518906
В капусте принесли?
Аноним 05/10/19 Суб 16:30:24 518969436
>>518887
Это с учетом подземных вод?
Аноним 05/10/19 Суб 16:59:53 518973437
Почему вокруг Земли нету мелких каменных спутников? Почему система Плутон-Харон сохранила мелкие луны вроде Гидры и Стикса, а Земля и Луна раскидали всё к хуям?
Аноним 05/10/19 Суб 17:47:51 518985438
>>518969
У Европы - объем воды при глубине океана в 100 километров при разбросе оценки в 70-180 км. То есть вероятно ее больше, но с меньшей вероятностью может быть и несколько меньше.
У Земли - вся вода начиная с грунтовых вод и выше.
>This sphere includes all of the water in the oceans, ice caps, lakes, rivers, groundwater, atmospheric water, and even the water in you, your dog, and your tomato plant
Количество воды в мантии, насколько я понимаю, не учитывается, поскольку оно неизвестно.
Аноним 05/10/19 Суб 19:01:51 519003439
>>518973
с земной Луной, вообще, дохренища неясного, и появление, и отношение масс, и период вращения, и состав.
Аноним 05/10/19 Суб 19:03:24 519004440
Аноним 05/10/19 Суб 20:21:54 519013441
Аноним 06/10/19 Вск 02:19:35 519056442
>>518973
> а Земля и Луна раскидали всё к хуям?
Скорее всего это раскидала Юпитер и Сатурн когда были гораздо ближе к Солнцу.
Аноним 06/10/19 Вск 09:39:01 519097443
>>518557
>данный аппарат "подхватывал" бы их
чтобы он не разбился об "похватываемые" грузы, эти грузы надо разогнать до орбитальной скорости, а если они уже разогнаны до орбитальной скорости, то нахрен этот аппарат тогда нужен?
Аноним 06/10/19 Вск 16:08:15 519174444
>>517528
Эта хуйня уже взорвалась, скорее всего. Свет никак дойти только не может.
Аноним 06/10/19 Вск 16:14:42 519175445
10682902-3x2-70[...].jpg (51Кб, 700x467)
700x467
Наверняка уже обсуждалось, но все же - а что если построить на орбите большой рельсотрон и запускать из него АМС или корабли в вакууме? Какие скорости будут? Или и людям, и технике пиздец будет от моментального ускорения? А все-таки - какие скорости? Можно как то с пользой это применить?
Аноним 06/10/19 Вск 16:21:00 519180446
>>519175
Если на орбите, то сам рельсотрон будет всякий раз улетать в другую сторону, лучше на Луне. Заодно можно в кратер поместить, где Солнца нет. Ускорение можно сделать мягче, скоростей можно добиваться очень больших. Можно использовать, например, для доставки продукции, произведённой на Луне, на Землю, или для вывода компонент космических кораблей, построенных на Луне. Но такая штука может потенциально стать серьёзным оружием.
Аноним 06/10/19 Вск 17:33:24 519185447
>>519180
>потенциально стать серьёзным оружием.
И может, и станет. Импульс можно и ионниками постепенно погасить. А так да, шмалядло которое пихает тонны со скоростью больше 4км/с это не только пушка лучше имеющихся на вооружении, но и двигатель даже неплохой.
Аноним 06/10/19 Вск 17:52:05 519186448
>>519175
Рельсотроны и на Земле хуево работают, требуя гигантского количества энергии и частой замены рельс, которые уже после пяти-десяти выстрелов приходят в полную непригодность. В данный момент нет нужных материалов, чтобы рельсотроны были чем-то практичным.
Аноним 06/10/19 Вск 17:54:50 519188449
>>519175
объясни целесообразность и экономическую выгоду
Аноним 06/10/19 Вск 18:05:39 519193450
>>519188
Ты дурак? Он об этом же и спрашивает. Ты на любой вопрос отвечаешь заданием этого же вопроса?
Аноним 06/10/19 Вск 18:35:50 519197451
1400322399634.jpg (48Кб, 800x450)
800x450
Почему видима неосвещенная сторона Луны?
Это как вообще блядь?
Я что, вижу отраженный свет звезд? Отраженный от Земли свет?
Или лунная поверхность сама излучает в видимом спектре?
Что это за хуйня вообще блядь, думаю уже 3-й день как это вообще возможно
Аноним 06/10/19 Вск 18:44:05 519199452
>>519197
>Отраженный от Земли свет?
Это.
Аноним 06/10/19 Вск 19:54:45 519213453
>>514328
какие технологии.
-во-первых российский робот Федор ананцефал. У него нет мозга и вообще он не робот , а аватар, теле-зонд. в РФ никаких роботов вообще нет, для этого нужны всякие си программисты с нейронными сетями, а в РФ таких людей нет.
-во вторых даже если рассматривать не роботов, а управляемые теле-зонды типа Федора, напомню что он не может даже удерживать в руке предмет, ему приклеивают предмет к руке изолентой левые люди.
-немецкий Саймон уже поумнее, но для безопасности у него нет рук, чтоб он вдруг не построил третий рейх.
-американцы запускают всяких полуроботов, типа марсоходов, но это капля в море
Аноним 06/10/19 Вск 20:11:10 519214454
>>518832
может в сша решили прикормить впк фирмы. а может они просчитали, что для достижения превосходства в текущей ситуации необходимы малозаметные ф35, а не 33000 ф15.

может у них нет авианосцев или аэродромов в ключевых местах, под 33 000. может они считают что эти 33 000 бесполезны, так как будут эффективно сбиваться пво, и не смогут выполнить сша-доктрину(разнести пво, а потом господствуя в воздухе разнести фронты)
Аноним 06/10/19 Вск 20:19:05 519219455
>>518893
Ты ебнутый,лол? У нее ядро есть,такая же железяка как у земли.
Аноним 06/10/19 Вск 21:17:19 519228456
Помню читал, что американцы рейлганом разогнали болванку до 7 км/с. Вроде бы это первая космическая. Значит можно огромной рельсой разгонять космический аппарат хотя бы до условных 5 км/с, а довыводить на орбиту уже собственными двигателями космолета?
Аноним 06/10/19 Вск 21:19:49 519229457
>>519228
ускорение слишком высокое будет, никакие механизмы не выдержат
а для норм ускорения нужна пушка в километры длинной
Аноним 06/10/19 Вск 21:40:25 519233458
>>519228
Твоя болванка сгорит уже через секунду, а пепел затормозится и упадет.

На уровне моря с космическими скоростями летать вообще нельзя, слишком сильно растет сопротивление (квадратично от скорости) и нагрев (от кубической зависимости от скорости для теплопроводности до восьмой степени для излучения).

>>519229
Километры это ты еще мало взял, даже если со 100g ускоряться, то процесс разгона займет 7 секунд, а пройденный путь составит 24,5 км. Если запускать людей, то там ну самый предел это 10g, и пушка вырастет еще в 10 раз, до 245 километров.
Аноним 06/10/19 Вск 21:54:17 519235459
Действительно ли земля может стать такой же как венера ?
Аноним 06/10/19 Вск 21:58:34 519236460
>>519233
>Твоя болванка сгорит уже через секунду, а пепел затормозится и упадет.
Так у них ничего не сгорало, они даже по мишеням стреляли.
Аноним 06/10/19 Вск 22:06:15 519237461
image.png (1617Кб, 1260x910)
1260x910
Аноним 06/10/19 Вск 22:09:03 519239462
Аноним 06/10/19 Вск 22:09:32 519240463
>>519236
Неси ссылку, в которой рейлган осилил 7 км/с, да еще и по мишеням. Подозреваю, что по факту окажется, что и не в лотерею, а в преферанс, и не выиграл, а проиграл.
Аноним 06/10/19 Вск 22:15:26 519241464
>>519235
Нет, у нас ни такого количества углерода нет, ни Солнце так сильно не печет в данный момент, через пару миллиардов лет другой разговор будет. На градусов 20 от современной температуры еще можно поднять, но венерианского климата на Земле не будет.
Аноним 06/10/19 Вск 22:18:05 519242465
>>519219
Схуяли железяка? Ледышка.
Аноним 06/10/19 Вск 22:21:15 519244466
>>519242
Схуяли ледышка то? Каким образом? От приливного захвата бы давно расстаяла,ядро как минимум каменное должно быть.
Аноним 06/10/19 Вск 22:22:27 519245467
image.png (2232Кб, 1280x960)
1280x960
Аноним 06/10/19 Вск 22:26:22 519246468
Сколько воды надо вылить в космос, чтобы образовался стабильный астероид исключительно из жидкой воды и водяного пара? Температура за счет солнечного излучения ровно 20 градусив, выливали аккуратно, так что гравитационного разогрева нет, вращение мегакапли - оборот за 5 часов, чтобы не было бешеной конвекции. (Под стабильностью я понимаю очень медленную потерю атмосферы, скажем не более 0,0001% за земной год).
Аноним 06/10/19 Вск 22:52:33 519253469
>>519246
Как минимум сферу радиусом с половину земного, я полагаю. Луна точно слишком маленькая, чтобы удержать жидкую воду в любом количестве, а Марс уже точно достаточно большой (но он гораздо плотнее воды).

Кстати, в центре все равно лёд будет, а не жидкая вода.
Аноним 06/10/19 Вск 22:53:37 519254470
>>519244
Она частично расстаяла, но вода под давлением превращается в лед, не забывай.
Аноним 06/10/19 Вск 23:00:32 519255471
>>519253
>Кстати, в центре все равно лёд будет, а не жидкая вода.
Хм, но ведь лед менее плотен чем вода и должен всплывать. Будет ли там бурбулятор из льда и воды околонулевой температуры в центре?

И будут ли недра без собственного источника тепла вообще охлаждаться? Снаружи-то везде теплая вода, да еще и атмосфера из водяного пара должна давать парниковый эффект, нет?
Аноним 06/10/19 Вск 23:03:49 519256472
>>519255
Не путай то, что по речкам плавает, с твердой водой в целом. Льда куча видов, и те, которые под высоким давлением и температурой выше 0°С существуют, более плотные, чем вода — там кристаллическая решетка более плотно упакована.
Аноним 06/10/19 Вск 23:05:19 519257473
>>519255
Лед, который под давлением, имеет другую плотность.
Аноним 06/10/19 Вск 23:09:14 519258474
>>519256
>>519257
Да, судя по тому, что давление в центре Земли на три порядка выше чем то, что нужно для такого льда, и в водном астероиде давления хватит.
Аноним 06/10/19 Вск 23:15:15 519261475
>>519258
А что, хочешь построить такую планету? Воду лучше налить на Марс.
Аноним 06/10/19 Вск 23:20:51 519263476
Аноним 06/10/19 Вск 23:21:45 519264477
>>519261
Ну это будет Земля на минималках, скукота. Вот чисто водяная планета будет чем-то новым. Прилетят инопланетяне - то-то удивятся.

Кстати, вот еще подумал: если есть ядро из плотного холодного льда (3, 9), то на него может намерзать обычный лед-1. Он хоть и менее плотный, чем вода, но всплывать не будет, потому что заякорится за ядро. То есть, планета в итоге будет иметь здоровое ледяное ядро, сверху океан жидкой воды, и потом уже атмосферу из водяного пара. Похоже на правду?
Аноним 06/10/19 Вск 23:22:01 519265478
>>519263
Чел, это просто картинка.
Аноним 06/10/19 Вск 23:22:46 519266479
>>519264
Да, так и будет. Но жизни на ней не будет.
Аноним 06/10/19 Вск 23:22:52 519267480
>>519264
Бляить,вы вообще понимаете,что такого быть в принципе не может?
Или сюда челы из зог протекли
Аноним 06/10/19 Вск 23:24:23 519268481
>>519266
Ну это понятно. По условиям же только вода (дистиллированная!).
Аноним 06/10/19 Вск 23:25:55 519269482
>>519267
Считай, что это что-то вроде огурцеводства, только без графона и с более гибкими правилами. (И без огурцов).
Аноним 06/10/19 Вск 23:35:06 519271483
>>519265
Масса и радиус Европы отлично известны. Отсюда находится плотность, которая в разы больше той, какая была бы от чистой воды. Наверняка при построении "картинки" были использованы и куда более хитрые выводы, но уж этот может любой школьник повторить.
Аноним 06/10/19 Вск 23:41:11 519273484
>>519265
Это хорошая гипотеза, лучше которой пока нет. Доживи до Европы Клиппера и миссии JUICE, узнаешь получше.
Аноним 07/10/19 Пнд 00:04:34 519275485
i.jpeg (13Кб, 290x180)
290x180
Задал Пашке вопрос про Вселенную на стриме, он отправил меня в пешее на другой свой канал мистики. Настораживает.
Аноним 07/10/19 Пнд 00:21:07 519277486
>>519275
Тут задавай, чо ты каких-то хохлов смотришь. Я рыцарь ордена Ганимеда, знаю ответы на все вопросы мира.
Аноним 07/10/19 Пнд 00:30:27 519278487
>>519277
Мне сама интерпретация от его дица была интересна, так как он оче умный персонаж.
Аноним 07/10/19 Пнд 00:44:37 519283488
>>519278
От нас ты че хочешь?
Аноним 07/10/19 Пнд 01:09:00 519290489
>>519235
Такой же как Венера стать не может, но теплее стать может. Смотри, разница в глобальных средних температурах между полностью ледниковыми периодами и безледниковыми оценивается примерно в 10º C. Что это значит? Когда жили динозавры средняя температура Земли была выше, чем в настоящее время, хотя Солнце светило слабее.
Сейчас ледниковый период, единственное не ясно заканчивается он или продолжается.

Аноним 07/10/19 Пнд 01:34:17 519297490
>>514000 (OP)
Оснащались ли американские космические корабли системой самоуничтожеения при приземлении на территории враждебного государства?
Аноним 07/10/19 Пнд 01:54:56 519299491
>>519297
Американцы о таком не упоминают. Думаю, что у них такого не было, такие штуки были характерны для параноидального СССР.
Аноним 07/10/19 Пнд 02:00:33 519301492
>>519290
В данный момент на Земле очевидно присутствуют ледниковые щиты и по прогнозам в ближайшие десятки тысяч лет они не растают, так что конец оледенения еще не близко. В нормальное время вне ледниковых периодов на планете вообще фактически не бывает многолетнего льда (только в горах в высоких широтах).
Аноним 07/10/19 Пнд 03:57:14 519309493
>>519301
>и по прогнозам в ближайшие десятки тысяч лет они не растают,
Ну, тут можно долго ломать копья. Одни утверждают, что началось потепление из-за антропогенного фактора другие говорят что никакого потепления нет и антропогенный фактор это хуита.
Аноним 07/10/19 Пнд 04:12:37 519311494
>>519297
Лол, конечно же нет.
Аноним 07/10/19 Пнд 04:14:31 519312495
>>519275
Кто такой? Какую ученую степень имеет и в каком научном центре работает?
Аноним 07/10/19 Пнд 04:16:53 519313496
>>519273
Два чаю, смысла гадать сейчас нет. Надо отправлять АМС и изучать.
Аноним 07/10/19 Пнд 04:18:02 519314497
>>519241
>Нет, у нас ни такого количества углерода нет
А сколько у насс тонн углерода в биосфере?
Аноним 07/10/19 Пнд 04:19:14 519315498
>>519236
Сгорит, точно сгорит. В плотных слоях атмосферы полет с такой скоростью моментально все нахуй испарит.
Аноним 07/10/19 Пнд 04:20:02 519316499
>>519213
>во-первых российский робот Федор ананцефал
Сделал мою ночь.
Аноним 07/10/19 Пнд 04:20:48 519318500
>>519315
Зависит от формы болванки, материала, вращения. Заострённая хуйня войдет как масло
Аноним 07/10/19 Пнд 04:21:42 519319501
>>519175
На орбите смысла нет, ибо отдача. А вот на космических телах не имеющих атмосферы почему бы и нет?
Тред тупых вопросов №114 Zeldovich edition Аноним 07/10/19 Пнд 04:22:14 519320502
3462.jpg (12Кб, 179x230)
179x230
Krupnomasshtabn[...].jpg (821Кб, 1024x768)
1024x768
15661134635360.jpg (57Кб, 1024x567)
1024x567
Тред вопросов о жизни, Вселенной и всём таком.

Спрашиваем то, за что в других местах выдают путёвку в биореактор. Здесь анонимные учёные мирового уровня критически рассмотрят любые гениальные идеи и нарисованные в Paint схемы.

Предыдущий тут: >>514000 (OP)
https://2ch.hk/spc/res/514000.html

Q: Можно быстрее?
A: Можно упасть в Пузырь Алькубьерре, NASA уже почти надула его (это шутка).

Q: Я начитался охуительных историй про уфологию, че делать, нам жопа?
A: Да, тебе жопа, можешь сгонять в зогач или куда оттуда пошлют.

Q: Что будет с человеком в вакууме без скафандра / если он упадет на черную дыру / попробует ступить на поверхность газового гиганта/солнца?
A: Он умрёт.

Q: Почему бы не привязать ракету к воздушному шару или стартовать с горы?
A: Космос - это не как высоко, а как быстро, большая часть энергии ракеты уходит на разгон вбок.
Подробнее тут https://what-if.xkcd.com/58/ (английский) https://chtoes.li/orbital-speed/ (перевод)
Аноним 07/10/19 Пнд 04:23:19 519322503
Аноним 07/10/19 Пнд 04:43:50 519325504
>>518947
Вроде бы нет. https://www.youtube.com/watch?v=u4RNGRyzt10
>Ведь у нейтронной звезды почти уже предельно минимальный радиус
Да, для ее массы. Кварковые звезды по идее должны быть плотнее, но как ты сам и сказал, они на порядок меньше размером.
Аноним 07/10/19 Пнд 09:26:30 519354505
Аноним 07/10/19 Пнд 10:19:27 519359506
>>519320
Ебанный стыд...
Во-первых, Алькубьерре.
Во-вторых, не упасть, а создавать вокруг корабля изнутри (иначе кина не будет).
В-третьих, НАСА искривляет пространство на десятимиллионную часть, контролируя это сверхточными интерферометрами, до самого варп-привода здесь - как до Антарктиды раком.
Аноним 07/10/19 Пнд 13:35:31 519402507
>>519309
Третьи говорят, что дальше на ожидает похолодание.
1 01/01/20 Срд 15:20:58 541701508
1
Настройки X
Ответить в тред X
15000 [S]
Макс объем: 40Mб, макс кол-во файлов: 4
Кликни/брось файл/ctrl-v
Стикеры X
Избранное / Топ тредов